Как найти производную сложной функции примеры: Производные сложных функций, основные формулы и примеры решений

Содержание

Производная сложной функции. Примеры решений

На данном уроке мы научимся находить производную сложной функции. Урок является логическим продолжением занятия Как найти производную?, на котором мы разобрали простейшие производные, а также познакомились с правилами дифференцирования и некоторыми техническими приемами нахождения производных. Таким образом, если с производными функций у Вас не очень или какие-нибудь моменты данной статьи будут не совсем понятны, то сначала ознакомьтесь с вышеуказанным уроком. Пожалуйста, настройтесь на серьезный лад – материал не из простых, но я все-таки постараюсь  изложить его просто и доступно.

На практике с производной сложной функции приходится сталкиваться очень часто, я бы даже сказал, почти всегда, когда Вам даны задания на нахождение производных.

Смотрим в таблицу на правило (№5) дифференцирования сложной функции:

 

Разбираемся. Прежде всего, обратим внимание на запись  . Здесь у нас две функции –   и  , причем функция  , образно говоря, вложена в функцию  . Функция такого вида (когда одна функция вложена в другую) и называется сложной функцией.

Функцию   я буду называть внешней функцией, а функцию   – внутренней (или вложенной) функцией.

! Данные определения не являются теоретическими и не должны фигурировать в чистовом оформлении заданий. Я применяю неформальные выражения «внешняя функция», «внутренняя» функция только для того, чтобы Вам легче было понять материал.

Для того, чтобы прояснить ситуацию, рассмотрим:

Пример 1

Найти производную функции 

Под синусом у нас находится не просто буква «икс», а целое выражение  , поэтому найти производную сразу по таблице не получится. Также мы замечаем, что здесь невозможно применить первые четыре правила, вроде бы есть разность, но дело в том, что «разрывать на части» синус нельзя: 

В данном примере уже из моих объяснений интуитивно понятно, что функция   – это сложная функция, причем многочлен   является внутренней функцией (вложением), а   – внешней функцией.

Первый шаг, который нужно выполнить при нахождении производной сложной функции состоит в том, чтобы разобраться, какая функция является внутренней, а какая – внешней.

В случае простых примеров вроде   понятно, что под синус вложен многочлен  . А как же быть, если всё не очевидно? Как точно определить, какая функция является внешней, а какая внутренней? Для этого я предлагаю использовать следующий прием, который можно проводить мысленно или на черновике.

Представим, что нам нужно вычислить на калькуляторе значение выражения   при   (вместо единицы может быть любое число).

Что мы вычислим в первую очередь? В первую очередь нужно будет выполнить следующее действие:  , поэтому многочлен   и будет внутренней функцией  :   Во вторую очередь нужно будет найти  , поэтому синус – будет внешней функцией: После того, как  мы РАЗОБРАЛИСЬ с внутренней и внешней функциями самое время применить правило дифференцирования сложной функции  .

Начинаем решать. С урока Как найти производную?

 мы помним, что оформление решения любой производной всегда начинается так – заключаем выражение в скобки и ставим справа вверху штрих:

Сначала находим производную внешней функции   (синуса), смотрим на таблицу производных элементарных функций и замечаем, что  . Все табличные формулы применимы и в том, случае, если «икс» заменить сложным выражением, в данном случае:

Обратите внимание, что внутренняя функция   не изменилась, её мы не трогаем.

Ну и совершенно очевидно, что 

Результат применения формулы   в чистовом оформлении выглядит так:

Далее мы берем производную внутренней функции, она очень простая:

Постоянный множитель обычно выносят в начало выражения:

Готово

Если осталось какое-либо недопонимание, перепишите решение на бумагу и еще раз прочитайте объяснения.

Пример 2

Найти производную функции 

Это пример для самостоятельного решения (ответ в конце урока).

Пример 3

Найти производную функции 

Как всегда записываем:

Разбираемся, где у нас внешняя функция, а где внутренняя. Для этого пробуем (мысленно или на черновике) вычислить значение выражения   при  . Что нужно выполнить в первую очередь? В первую очередь нужно сосчитать чему равно основание:  , значит, многочлен   – и есть внутренняя функция: И, только потом выполняется возведение в степень  , следовательно, степенная функция – это внешняя функция: Согласно формуле  , сначала нужно найти производную от внешней функции, в данном случае, от степени. Разыскиваем в таблице нужную формулу:  . Повторяем еще раз: 

любая табличная формула справедлива не только для «икс», но и для сложного выражения. Таким образом, результат применения правила дифференцирования сложной функции    следующий:

Снова подчеркиваю, что когда мы берем производную от внешней функции  , внутренняя функция   у нас не меняется: Теперь осталось найти совсем простую производную от внутренней функции и немного «причесать» результат:

Готово.

Пример 4

Найти производную функции 

Это пример для самостоятельного решения (ответ в конце урока).

Для закрепления понимания производной сложной функции приведу пример без комментариев, попробуйте самостоятельно разобраться, порассуждать, где внешняя и где внутренняя функция, почему задания решены именно так?

Пример 5

а) Найти производную функции 

б) Найти производную функции 

Пример 6

Найти производную функции 

Здесь у нас корень, а для того, чтобы продифференцировать корень, его нужно представить в виде степени  . Таким образом, сначала приводим функцию в надлежащий для дифференцирования вид:

Анализируя функцию, приходим к выводу, что сумма трех слагаемых – это внутренняя функция, а возведение в степень – внешняя функция.

Применяем правило дифференцирования сложной функции  :

Степень снова представляем в виде радикала (корня), а для производной внутренней функции применяем простое правило дифференцирования суммы:

Готово. Можно еще в скобках привести выражение к общему знаменателю и записать всё одной дробью. Красиво, конечно, но когда получаются громоздкие длинные производные – лучше этого не делать (легко запутаться, допустить ненужную ошибку, да и преподавателю будет неудобно проверять).

Пример 7

Найти производную функции 

Это пример для самостоятельного решения (ответ в конце урока).

Интересно отметить, что иногда вместо правила дифференцирования сложной функции можно использовать правило дифференцирования частного  , но такое решение будет выглядеть как извращение забавно. Вот характерный пример:

Пример 8

Найти производную функции 

Здесь можно использовать правило дифференцирования частного  , но гораздо выгоднее найти производную через правило дифференцирования сложной функции:

Подготавливаем функцию для дифференцирования – выносим минус за знак производной, а косинус поднимаем в числитель:

Косинус – внутренняя функция, возведение в степень – внешняя функция.   Используем наше правило  :

Находим производную внутренней функции, косинус сбрасываем обратно вниз:

Готово. В рассмотренном примере важно не запутаться в знаках. Кстати, попробуйте решить его с помощью правила  , ответы должны совпасть.

Пример 9

Найти производную функции 

Это пример для самостоятельного решения (ответ в конце урока).

До сих пор мы рассматривали случаи, когда у нас в сложной функции было только одно вложение. В практических же заданиях часто можно встретить производные, где, как матрешки, одна в другую,  вложены сразу 3, а то и 4-5 функций.

Пример 10

Найти производную функции 

Разбираемся во вложениях этой функции. Пробуем вычислить выражение   с помощью подопытного значения  . Как бы мы считали на калькуляторе?

Сначала нужно найти  , значит, арксинус – самое глубокое вложение:

Затем этот арксинус единицы следует возвести в квадрат  :

И, наконец, семерку возводим в степень  : То есть, в данном примере у нас три разные функции и два вложения, при этом, самой внутренней функцией является арксинус, а самой внешней функцией – показательная функция.

Начинаем решать

Согласно правилу   сначала нужно взять производную от внешней функции. Смотрим в таблицу производных и находим производную показательной функции:  Единственное отличие – вместо «икс» у нас сложное выражение   , что не отменяет справедливость данной формулы. Итак, результат применения правила дифференцирования сложной функции    следующий:

Под штрихом у нас снова сложная функция! Но она уже проще. Легко убедиться, что внутренняя функция – арксинус, внешняя функция – степень. Согласно правилу дифференцирования сложной функции сначала нужно взять производную от степени:

Теперь все просто, находим по таблице производную арксинуса и немного «причесываем» выражение:

Готово.

Пример 11

Найти производную функции 

Это пример для самостоятельного решения (ответ в конце урока).

На практике правило дифференцирования сложной функции почти всегда применяется в комбинации с остальными правилами дифференцирования.

Пример 12

Найти производную функции 

Сначала используем правило дифференцирования суммы  , заодно в первом слагаемом выносим постоянный множитель за знак производной по правилу  :

В обоих слагаемых под штрихами у нас находится произведение функций, следовательно, нужно дважды применить правило  :

Замечаем, что под некоторыми штрихами у нас находятся сложные функции  ,  . Каламбур, но это простейшие из сложных функций, и при определенном опыте решения производных Вы будете легко находить их устно. А пока запишем подробно, согласно правилу  , получаем:

Готово.

! Обратите внимание на приоритет (порядок) применения правил: правило дифференцирования сложной функции применяется в последнюю очередь.

Пример 13

Найти производную функции 

Это пример для самостоятельного решения (ответ в конце урока).

Пожалуй, хватит на сегодня. Хочется еще привести пример с дробью и сложной функцией, но такой пример принципиально ничем не отличается от двух последних заданий, единственное отличие – вместо правила   применяем правило  .

Для закрепления темы рекомендую статью Сложные производные. Логарифмическая производная. Помимо рассмотрения дополнительных примеров, есть и новый материал! После изучения третьего урока вы будете очень уверенно себя чувствовать в ходе дальнейшего изучения математического анализа. Если задания покажутся слишком трудными (у всех разный уровень подготовки), то сначала посетите страницу Простейшие типовые задачи с производной, там рассмотрено ещё порядка 15-ти производных.

Желаю успехов!

Ответы:

Пример 2: 

Пример 4:  Указание: перед дифференцированием необходимо перенести степень наверх, сменив у показателя знак .

Пример 7: 

Пример 9: 

Пример 11: 

Пример 13: 

Математика онлайн

Решение математики онлайн

Math34.biz – это современный способ решения математики, в том числе для сравнения самостоятельных решений с машинными вычислениями.

Пользование сервисом удобно и понятно каждому человеку, попавшему на сайт впервые. Сразу выбираете нужный калькулятор, вводите необходимые данные по вашей задаче и нажимаете кнопку «Решение». За считанные секунды ответ готов.

Чтобы не возникало трудностей с вводом данных, мы подготовили специальную статью Как вводить данные? Помимо правил написания формул и чисел, в ней вы можете увидеть, как правильно вводятся различные константы и математические функции.

О калькуляторах

По мере возможности добавляются новые математические калькуляторы. На сегодняшний день их более 85.

Если не удалось найти нужный калькулятор, которым может быть решена ваша математическая задача, или есть предложение по улучшению имеющегося калькулятора, пожалуйста, сообщите об этом на почту [email protected]

Преимущества

1. Бесплатно
Решение математики онлайн не будет вам стоить ни копейки. Наш сервис абсолютно бесплатный и доступен любому пользователю интернета.

2. Без регистрации
Для пользования калькуляторами не требуется регистрации на сайте, отнимая время на заполнение почтовых ящиков и других личных данных.

3. Подробные решения
На многие задачи вы получите пошаговый развернутый ответ, что позволяет понять, каким образом было получено решение задачи.

4. Разные способы решения задач
Для популярных калькуляторов доступны разные методы решения задач, если они применимы, что позволяет, во-первых, лучше понять, как решается задача известным вам способом, а, во-вторых, научиться решать ту же самую задачу альтернативными методами.

5. Точность вычислений
В полученном ответе не приходится сомневаться, ведь мощная система расчета обеспечивает высокую точность при решении математических задач онлайн.

Однако, мы не исключаем возможность каких-либо ошибок, ведь известно, что алгоритмы пишутся хотя и очень умными, но всё же людьми. В случае обнаружения ошибки, пожалуйста, не поленитесь и сообщите нам о ней.

Производная сложной функции.

Инструкционная карта № 19

Тақырыбы/ Тема: «Производная сложной функции».

Мақсаты/ Цель:

1.Обеспечить усвоение учащимися умения применять формулы дифференцирования сложной функции и вычисления этой производной при решении упражнений и заданий.

2. При решении упражнений, развивать у учащихся умения выделять главное, существенное в изучаемом материале, обучить умению рационально находить правильное решение изучаемого вопроса.

3. Создать условие для развития коммутативно-творческих умений: не шаблонно подходить решению разнообразных задач.

Теоретический материал:

На практике с производной сложной функции приходится сталкиваться очень часто, почти всегда, когда Вам даны задания на нахождение производных.

Смотрим на правило дифференцирования сложной функции: 

Разбираемся. Прежде всего, обратим внимание на запись . Здесь у нас две функции –  и , причем функция , образно говоря, вложена в функцию . Функция такого вида (когда одна функция вложена в другую) и называется сложной функцией.

Функцию   будем называть внешней функцией, а функцию  – внутренней (или вложенной) функцией.

! Данные определения не являются теоретическими и не должны фигурировать в чистовом оформлении заданий. Эти неформальные выражения «внешняя функция», «внутренняя» функция только для того, чтобы Вам легче было понять материал.

Для того, чтобы прояснить ситуацию, рассмотрим:

Пример 1. Найти производную функции: 

Под синусом у нас находится не просто буква «икс», а целое выражение , поэтому найти производную сразу по таблице не получится. Также мы замечаем, что здесь невозможно применить первые четыре правила, вроде бы есть разность, но дело в том, что «разрывать на части» синус нельзя: 

В данном примере уже из моих объяснений интуитивно понятно, что функция  – это сложная функция, причем многочлен  является внутренней функцией (вложением), а  – внешней функцией.

Первый шаг, который нужно выполнить при нахождении производной сложной функции состоит в том, чтобы разобраться, какая функция является внутренней, а какая – внешней.

В случае простых примеров вроде  понятно, что под синус вложен многочлен . А как же быть, если всё не очевидно? Как точно определить, какая функция является внешней, а какая внутренней? Для этого будем использовать следующий прием, который можно проводить мысленно или на черновике.

Представим, что нам нужно вычислить на калькуляторе значение выражения  при  (вместо единицы может быть любое число).

Что мы вычислим в первую очередь? В первую очередь нужно будет выполнить следующее действие: , поэтому многочлен  и будет внутренней функцией :
 
Во вторую очередь нужно будет найти , поэтому синус – будет внешней функцией:

После того, как  мы РАЗОБРАЛИСЬ с внутренней и внешней функциями самое время применить правило дифференцирования сложной функции .

Начинаем решать. Из предыдущего урока  мы помним, что оформление решения любой производной всегда начинается так – заключаем выражение в скобки и ставим справа вверху штрих:

Сначала находим производную внешней функции  (синуса), смотрим на таблицу производных элементарных функций и замечаем, что . Все табличные формулы применимы и в том, случае, если «икс» заменить сложным выражением, в данном случае:

Обратите внимание, что внутренняя функция  не изменилась, её мы не трогаем.

Ну и совершенно очевидно, что 

Результат применения формулы  в чистовом оформлении выглядит так:

Далее мы берем производную внутренней функции, она очень простая:

Постоянный множитель обычно выносят в начало выражения:
Готово.

Если осталось какое-либо недопонимание, перепишите решение на бумагу и еще раз прочитайте объяснения.

Пример 2. Найти производную функции:  у’=(cos2x)’=-sin2x(2x)’=-2sin2x.

Пример 3. Найти производную функции:  Как всегда записываем:

Разбираемся, где у нас внешняя функция, а где внутренняя. Для этого пробуем (мысленно или на черновике) вычислить значение выражения  при . Что нужно выполнить в первую очередь? В первую очередь нужно сосчитать чему равно основание: , значит, многочлен  – и есть внутренняя функция:


И, только потом выполняется возведение в степень , следовательно, степенная функция – это внешняя функция:

Согласно формуле , сначала нужно найти производную от внешней функции, в данном случае, от степени. Разыскиваем в таблице нужную формулу: . Повторяем еще раз: любая табличная формула справедлива не только для «икс», но и для сложного выражения. Таким образом, результат применения правила дифференцирования сложной функции   следующий:

Снова подчеркиваю, что когда мы берем производную от внешней функции , внутренняя функция  у нас не меняется:

Теперь осталось найти совсем простую производную от внутренней функции и немного «причесать» результат:

Готово.

Пример 4. Найти производную функции:  у’= -(х2-1)’= – = – .

Для закрепления понимания производной сложной функции приведу пример без комментариев, попробуйте самостоятельно разобраться, порассуждать, где внешняя и где внутренняя функция, почему задания решены именно так?

Пример 5 а) Найти производную функции: 

б) Найти производную функции: 

Пример 6. Найти производную функции: 

Здесь у нас корень, а для того, чтобы продифференцировать корень, его нужно представить в виде степени . Таким образом, сначала приводим функцию в надлежащий для дифференцирования вид:

Анализируя функцию, приходим к выводу, что сумма трех слагаемых – это внутренняя функция, а возведение в степень – внешняя функция. Применяем правило дифференцирования сложной функции :

Степень снова представляем в виде радикала (корня), а для производной внутренней функции применяем простое правило дифференцирования суммы:

Готово.

Можно еще в скобках привести выражение к общему знаменателю и записать всё одной дробью. Красиво, конечно, но когда получаются громоздкие длинные производные – лучше этого не делать (легко запутаться, допустить ненужную ошибку, да и преподавателю будет неудобно проверять).

Пример 7. Найти производную функции:

y’=(1+ = – .

Интересно отметить, что иногда вместо правила дифференцирования сложной функции можно использовать правило дифференцирования частного , но такое решение будет выглядеть как извращение необычно. Вот характерный пример:

Пример 8. Найти производную функции:

Здесь можно использовать правило дифференцирования частного , но гораздо выгоднее найти производную через правило дифференцирования сложной функции:

Подготавливаем функцию для дифференцирования – выносим минус за знак производной, а косинус поднимаем в числитель:

Косинус – внутренняя функция, возведение в степень – внешняя функция.  
Используем наше правило :

Находим производную внутренней функции, косинус сбрасываем обратно вниз:

Готово.

Пример 9. Найти производную функции: 

y’= – (arcos x)’ = – =.

До сих пор мы рассматривали случаи, когда у нас в сложной функции было только одно вложение. В практических же заданиях часто можно встретить производные, где, как матрешки, одна в другую,  вложены сразу 3, а то и 4-5 функций.

Пример 10. Найти производную функции: 

Разбираемся во вложениях этой функции. Пробуем вычислить выражение  с помощью подопытного значения . Как бы мы считали на калькуляторе?

Сначала нужно найти , значит, арксинус – самое глубокое вложение:

Затем этот арксинус единицы следует возвести в квадрат :

И, наконец, семерку возводим в степень :

То есть, в данном примере у нас три разные функции и два вложения, при этом, самой внутренней функцией является арксинус, а самой внешней функцией – показательная функция.

Начинаем решать

Согласно правилу  сначала нужно взять производную от внешней функции. Смотрим в таблицу производных и находим производную показательной функции:  Единственное отличие – вместо «икс» у нас сложное выражение  , что не отменяет справедливость данной формулы. Итак, результат применения правила дифференцирования сложной функции   следующий:

Под штрихом у нас снова сложная функция! Но она уже проще. Легко убедиться, что внутренняя функция – арксинус, внешняя функция – степень. Согласно правилу дифференцирования сложной функции сначала нужно взять производную от степени:

Теперь все просто, находим по таблице производную арксинуса и немного «причесываем» выражение:

Готово.

Пример 11. Найти производную функции: 

y’=2ln(2x-1)(ln(2x-1)’=2ln(2x-1)(2x-1)’ =2ln(2x-1)2=

На практике правило дифференцирования сложной функции почти всегда применяется в комбинации с остальными правилами дифференцирования.

Пример 12. Найти производную функции: 

Сначала используем правило дифференцирования суммы , заодно в первом слагаемом выносим постоянный множитель за знак производной по правилу :

В обоих слагаемых под штрихами у нас находится произведение функций, следовательно, нужно дважды применить правило :

Замечаем, что под некоторыми штрихами у нас находятся сложные функции , . Каламбур, но это простейшие из сложных функций, и при определенном опыте решения производных Вы будете легко находить их устно.
А пока запишем подробно, согласно правилу , получаем:

Готово.

! Обратите внимание на приоритет (порядок) применения правил: правило дифференцирования сложной функции применяется в последнюю очередь.

Пример 13 Найти производную функции: 

y’=( =

= + ctgx.

Практическая часть:

1 вариант

Найти производную сложной функции:

2 вариант

Найти производную сложной функции:

3 вариант

Найти производную сложной функции:

4 вариант

Найти производную сложной функции:

.

.

.

.

.

.

.

.

.

.

.

.

.

.

.

.

.

.

.

.

.

.

.

y=(x2-5x+8)6.

y= .

у = lg(5x2+1) .

y= .

3(e) .

y=ln .

y=tg(2x2+1) .

y=ln .

y=ln2(x2-1).

y=cos.

y=sin3mx .

y=arccos .

y=e, найти: y’().

y=sin, найти: y’().

y=3, найти: у’(1) .

у = (7-6х)12, найти: у'(1).

Контрольные вопросы:

  1. Дайте определение сложной функции.

  2. Как находится производная сложной функции? Пояснить ответ на примере.

Производные функций – Как найти производную Примеры решений


Подборка по базе: Характеристика поведения и инвестиционных решений субъектов ЭЭР , Система контроля реализации управленческих решений.pptx, Реферат Методы оптимальных решений (1).docx, Дескриптивная модель принятия решений. преимущества и ограничени, по кассе примеры.docx, 15 дерево решений.docx, НГУЭУ_Принципы и методы исследований и принятия решений_КР_Вариа, 13. Процессы принятия решений в организации.docx, 1.2 Примеры решения тестов.pdf, Линейная алгебра примеры.doc
  1   2   3   4   5   6   7   8 Как найти производную?
Примеры решений
Как найти производную, как взять производную На данном уроке мы научимся находить производные функций. Но перед изучением данной страницы я настоятельно рекомендую ознакомиться с методическим материалом Горячие формулы школьного курса математики. Справочное пособие можно открыть или закачать на странице Математические формулы и таблицы. Также оттуда нам потребуется Таблица производных, ее лучше распечатать, к ней часто придется обращаться, причем, не только сейчас, но ив оффлайне. Есть Приступим. У меня для Вас есть две новости хорошая и очень хорошая. Хорошая новость состоит в следующем чтобы научиться находить производные, совсем необязательно знать и понимать, что такое производная. Более того, определение производной функции, математический, физический, геометрический смысл производной целесообразнее переварить позже, поскольку качественная проработка теории, по моему мнению, требует изучения ряда других тема также некоторого практического опыта. И сейчас наша задача освоить эти самые производные технически. Очень хорошая новость состоит в том, что научиться брать производные не так сложно, существует довольно чёткий алгоритм решения (и объяснения) этого задания, интегралы или пределы, например, освоить труднее.
Советую следующий порядок изучения темы во-первых, эта статья. Затем нужно прочитать важнейший урок Производная сложной функции. Эти два базовых занятия позволят поднять Ваши навыки с полного нуля. Далее можно будет ознакомиться с более сложными производными в статье Сложные производные.
Логарифмическая производная. Если планка окажется слишком высока, то сначала прочитайте вещь Простейшие типовые задачи с производной. Помимо нового материала, на уроке рассмотрены другие, более простые типы производных, и есть прекрасная возможность улучшить свою технику дифференцирования. Кроме того, в контрольных работах почти всегда встречаются задания на нахождение производных функций, которые заданы неявно или параметрически. Такой урок тоже есть Производные неявных и параметрически заданных функций.
Я попытаюсь в доступной форме, шаг за шагом, научить Вас находить производные функций. Вся информация изложена подробно, простыми словами. Собственно, сразу рассмотрим пример:
Пример Найти производную функции Решение Это простейший пример, пожалуйста, найдите его в таблице производных элементарных функций. Теперь посмотрим на решение и проанализируем, что же произошло А произошла следующая вещь:
у нас была функция
, которая в результате решения превратилась в функцию Говоря совсем просто, для того чтобы найти производную функции, нужно по определенным правилам превратить её в другую функцию. Посмотрите еще раз на таблицу производных – там функции превращаются в другие функции. Единственным исключением является экспоненциальная функция
, которая
превращается сама в себя. Операция нахождения производной называется дифференцированием.
Обозначения: Производную обозначают или ВНИМАНИЕ, ВАЖНО Забыть поставить штрих (там, где надо, либо нарисовать лишний штрих (там, где не надо) – ГРУБАЯ ОШИБКА Функция и её производная – это две разные функции!
Вернемся к нашей таблице производных. Изданной таблицы желательно запомнить наизусть правила дифференцирования и производные некоторых элементарных функций, особенно:
производную константы, где – постоянное число;
производную степенной функции, в частности
,
, Зачем запоминать Данные знания являются элементарными знаниями о производных. И если Вы не сможете ответить преподавателю на вопрос Чему равна производная числа, то учеба в ВУЗе может для Вас закончиться (лично знаком с двумя реальными случаями из жизни. Кроме того, это наиболее распространенные формулы, которыми приходится пользоваться практически каждый раз, когда мы сталкиваемся с производными.
В реальности простые табличные примеры – редкость, обычно при нахождении производных сначала используются правила дифференцирования, а затем – таблица производных элементарных функций.
В этой связи переходим к рассмотрению правил дифференцирования


1) Постоянное число можно (и нужно) вынести за знак производной, где – постоянное число (константа)
Пример Найти производную функции Смотрим в таблицу производных. Производная косинуса там есть, ноу нас
Решаем:
Самое время использовать правило, выносим постоянный множитель за знак производной:
А теперь превращаем наш косинус по таблице:
Ну и результат желательно немного причесать – ставим минус на первое место, заодно избавляясь от скобок:
Готово.
2) Производная суммы равна сумме производных
Пример 3

Найти производную функции Решаем. Как Вы, наверное, уже заметили, первое действие, которое всегда выполняется при нахождении производной, состоит в том, что мы заключаем в скобки всё выражение и ставим штрих справа вверху:
Применяем второе правило:
Обратите внимание, что для дифференцирования все корни, степени нужно представить в виде
, а если они находятся в знаменателе, то переместить их вверх. Как это сделать – рассмотрено в моих методических материалах.
Теперь вспоминаем о первом правиле дифференцирования – постоянные множители (числа) выносим за знак производной Обычно входе решения эти два правила применяют одновременно чтобы не переписывать лишний раз длинное выражение).
Все функции, находящиеся под штрихами, являются элементарными табличными функциями, с помощью таблицы осуществляем превращение:
Можно всё оставить в таком виде, так как штрихов больше нет, и производная найдена. Тем не менее, подобные выражения обычно упрощают Все степени вида желательно снова представить в виде корней, степени с отрицательными показателями – сбросить в знаменатель. Хотя этого можно и не делать, ошибкой не будет.
Пример Найти производную функции Попробуйте решить данный пример самостоятельно (ответ в конце урока) Производная произведения функций
Вроде бы по аналогии напрашивается формула
…., но неожиданность состоит в том, что:
Эта необычное правило как, собственно, и другие следует из определения производной. Нос теорией мы пока повременим – сейчас важнее научиться решать:
Пример Найти производную функции Здесь у нас произведение двух функций, зависящих от . Сначала применяем наше странное правило, а затем превращаем функции по таблице производных:
Сложно? Вовсе нет, вполне доступно даже для чайника

Пример Найти производную функции В данной функции содержится сумма и произведение двух функций – квадратного трехчлена и логарифма Со школы мы помним, что умножение и деление имеют приоритет перед сложением и вычитанием.
Здесь всё также. СНАЧАЛА мы используем правило дифференцирования произведения:
Теперь для скобки используем два первых правила:
В результате применения правил дифференцирования под штрихами у нас остались только элементарные функции, по таблице производных превращаем их в другие функции:
Готово.
При определенном опыте нахождения производных, простые производные вроде необязательно расписывать так подробно. Вообще, они обычно решаются устно, и сразу записывается, что

Пример Найти производную функции Это пример для самостоятельного решения (ответ в конце урока) Производная частного функций
В потолке открылся люк, не пугайся, это глюк. А вот это вот суровая действительность:
Пример Найти производную функции Чего здесь только нет – сумма, разность, произведение, дробь. С чего бы начать Есть сомнения, нет сомнений, но, В ЛЮБОМ СЛУЧАЕ для начала рисуем скобочки и справа вверху ставим штрих:
Теперь смотрим на выражение в скобках, как бы его упростить В данном случае замечаем множитель, который согласно первому правилу целесообразно вынести за знак производной:
Заодно избавляемся от скобок в числителе, которые теперь ненужны. Вообще говоря, постоянные множители при нахождении производной можно и не выносить, нов этом случае они будут путаться под ногами, что загромождает и затрудняет решение.
Смотрим на наше выражение в скобках. У насесть сложение, вычитание и деление. Со школы мы помним, что деление выполняется в первую очередь. И здесь – сначала применяем правило дифференцирования частного:
Таким образом, наша страшная производная свелась к производным двух простых выражений. Применяем первое и второе правило, здесь это сделаем устно, надеюсь, Вы уже немного освоились в производных:
Штрихов больше нет, задание выполнено. На практике обычно (ноне всегда) ответ упрощают школьными методами Пример Найти производную функции Это пример для самостоятельного решения (ответ в конце урока).
Время от времени встречаются хитрые задачки:
Пример Найти производную функции Смотрим на данную функцию. Здесь снова дробь. Однако перед тем как использовать правило дифференцирования частного (а его можно использовать, всегда имеет смысл посмотреть, а нельзя ли упростить саму дробь, или вообще избавиться от нее?
Дело в том, что формула достаточно громоздка, и применять ее совсем не хочется. В данном случае можно почленно поделить числитель на знаменатель.
Преобразуем функцию Ну вот, совсем другое дело, теперь дифференцировать простои приятно:
Готово.
Пример Найти производную функции Здесь ситуация похожа, превратим нашу дробь в произведение, для этого поднимем экспоненту в числитель, сменив у показателя знак:
Произведение все-таки дифференцировать проще:
Пример Найти производную функции Это пример для самостоятельного решения (ответ в конце урока


5) Производная сложной функции
Данное правило также встречается очень часто. Но он м рассказать можно очень много, поэтому я создал отдельный урок на тему Производная сложной функции.
Желаю успехов!
Ответы:
Пример 4:
. Входе решения данного примера следует обратить внимание, на тот факт, что и
– постоянные числа, неважно чему они равны, важно, что это – константы. Поэтому
выносится за знак производной, а Пример 7: Пример 9: Пример 12:

Производная сложной функции. Примеры решений
На данном уроке мы научимся находить производную сложной функции. Урок является логическим продолжением занятия Как найти производную, на котором мы разобрали простейшие производные, а также познакомились с правилами дифференцирования и некоторыми техническими приемами нахождения производных. Таким образом, если с производными функций у Вас не очень или какие-нибудь моменты данной статьи будут не совсем понятны, то сначала ознакомьтесь с вышеуказанным уроком. Пожалуйста, настройтесь на серьезный лад – материал не из простых, ноя все-таки постараюсь изложить его простои доступно. На практике с производной сложной функции приходится сталкиваться очень часто, я бы даже сказал, почти всегда, когда Вам даны задания на нахождение производных.
Смотрим в таблицу направило) дифференцирования сложной функции:
Разбираемся. Прежде всего, обратим внимание на запись
. Здесь у нас две функции – и , причем функция , образно говоря, вложена в функцию . Функция такого вида (когда одна функция вложена в другую) и называется сложной функцией.
Функцию я буду называть внешней функцией, а функцию – внутренней (или вложенной) функцией.
! Данные определения не являются теоретическими и не должны фигурировать в чистовом оформлении заданий. Я применяю
неформальные выражения внешняя функция, внутренняя функция только для того, чтобы Вам легче было понять материал.
Для того, чтобы прояснить ситуацию, рассмотрим:
Пример Найти производную функции Под синусом у нас находится непросто буква икса целое выражение
, поэтому найти производную сразу по таблице не получится. Также мы замечаем, что здесь невозможно применить первые четыре правила, вроде бы есть разность, но дело в том, что разрывать на части синус нельзя В данном примере уже из моих объяснений интуитивно понятно, что функция
– это сложная функция, причем многочлен является внутренней функцией (вложением, а
– внешней функцией.
Первый шаг, который нужно выполнить при нахождении производной сложной функции состоит в том, чтобы разобраться, какая функция является внутренней, а какая – внешней.
В случае простых примеров вроде понятно, что под синус вложен многочлен
. А как же быть, если всё не очевидно Как точно определить, какая функция является внешней, а какая внутренней Для этого я предлагаю использовать следующий прием, который можно проводить мысленно или на черновике.
Представим, что нам нужно вычислить на калькуляторе значение выражения при
(вместо единицы может быть любое число).
Что мы вычислим в первую очередь В первую очередь нужно будет
выполнить следующее действие
, поэтому многочлен и будет внутренней функцией Во вторую очередь нужно будет найти
, поэтому синус – будет внешней функцией:
После того, как мы РАЗОБРАЛИСЬ с внутренней и внешней функциями самое время применить правило дифференцирования сложной функции Начинаем решать. Из урока Как найти производную мы помним, что оформление решения любой производной всегда начинается так – заключаем выражение в скобки и ставим справа вверху штрих:
Сначала находим производную внешней функции
(синуса, смотрим на таблицу производных элементарных функций и замечаем,
что
. Все табличные формулы применимы ив том, случае, если икс заменить сложным выражением, в данном случае:
Обратите внимание, что внутренняя функция не изменилась, её мы не трогаем Ну и совершенно очевидно, что Результат применения формулы в чистовом оформлении выглядит так:
Далее мы берем производную внутренней функции, она очень простая:
Постоянный множитель обычно выносят в начало выражения:
Готово
Если осталось какое-либо недопонимание, перепишите решение на бумагу и еще раз прочитайте объяснения.
Пример Найти производную функции Это пример для самостоятельного решения (ответ в конце урока).
Пример Найти производную функции Как всегда записываем:
Разбираемся, где у нас внешняя функция, а где внутренняя. Для этого пробуем (мысленно или на черновике) вычислить значение выражения при
. Что нужно выполнить в первую очередь В первую очередь нужно сосчитать чему равно основание


, значит, многочлен
– и есть внутренняя функция:
И, только потом выполняется возведение в степень , следовательно, степенная функция – это внешняя функция:
Согласно формуле
, сначала нужно найти производную от внешней функции, в данном случае, от степени. Разыскиваем в таблице нужную формулу
. Повторяем еще раз любая табличная формула справедлива не только для икс, но и для сложного выражения. Таким образом, результат применения правила дифференцирования сложной функции следующий:
Снова подчеркиваю, что когда мы берем производную от внешней функции
, внутренняя функция у нас не меняется:
Теперь осталось найти совсем простую производную от внутренней функции и немного причесать результат:
Готово.

Пример Найти производную функции Это пример для самостоятельного решения (ответ в конце урока).
Для закрепления понимания производной сложной функции приведу пример без комментариев, попробуйте самостоятельно разобраться, порассуждать, где внешняя и где внутренняя функция, почему задания решены именно так?
Пример а) Найти производную функции б) Найти производную функции Пример Найти производную функции Здесь у нас корень, а для того, чтобы продифференцировать корень, его нужно представить в виде степени
. Таким образом, сначала приводим функцию в надлежащий для дифференцирования вид:
Анализируя функцию, приходим к выводу, что сумма трех слагаемых

– это внутренняя функция, а возведение в степень – внешняя функция.
Применяем правило дифференцирования сложной функции
:
Степень снова представляем в виде радикала (корня, а для производной внутренней функции применяем простое правило дифференцирования суммы:
Готово. Можно еще в скобках привести выражение к общему знаменателю и записать всё одной дробью. Красиво, конечно, но когда получаются громоздкие длинные производные – лучше этого не делать (легко запутаться, допустить ненужную ошибку, да и преподавателю будет неудобно проверять).
Пример Найти производную функции Это пример для самостоятельного решения (ответ в конце урока

Интересно отметить, что иногда вместо правила дифференцирования сложной функции можно использовать правило дифференцирования частного
, но такое решение будет выглядеть как извращение необычно. Вот характерный пример:
Пример Найти производную функции Здесь можно использовать правило дифференцирования частного, но гораздо выгоднее найти производную через правило дифференцирования сложной функции:
Подготавливаем функцию для дифференцирования – выносим минус за знак производной, а косинус поднимаем в числитель:
Косинус – внутренняя функция, возведение в степень – внешняя функция. Используем наше правило Находим производную внутренней функции, косинус сбрасываем

обратно вниз:
Готово. В рассмотренном примере важно не запутаться в знаках. Кстати, попробуйте решить его с помощью правила
, ответы должны совпасть.
Пример Найти производную функции Это пример для самостоятельного решения (ответ в конце урока).
До сих пор мы рассматривали случаи, когда у нас в сложной функции было только одно вложение. В практических же заданиях часто можно встретить производные, где, как матрешки, одна в другую, вложены сразу 3, а то и 4-5 функций.
Пример Найти производную функции Разбираемся во вложениях этой функции. Пробуем вычислить выражение с помощью подопытного значения
. Как бы мы считали на калькуляторе?
Сначала нужно найти
, значит, арксинус – самое глубокое вложение

Затем этот арксинус единицы следует возвестив квадрат И, наконец, семерку возводим в степень То есть, в данном примере у нас три разные функции и два вложения, при этом, самой внутренней функцией является арксинуса самой внешней функцией – показательная функция.
Начинаем решать
Согласно правилу сначала нужно взять производную от внешней функции. Смотрим в таблицу производных и находим производную показательной функции
Единственное отличие – вместо иксу нас сложное выражение
, что не отменяет справедливость данной формулы. Итак, результат применения правила дифференцирования сложной функции следующий:
Под штрихом у нас снова сложная функция Но она уже проще. Легко убедиться, что внутренняя функция – арксинус, внешняя функция – степень. Согласно правилу дифференцирования сложной функции

сначала нужно взять производную от степени:
Теперь все просто, находим по таблице производную арксинуса и немного причесываем выражение:
Готово.
Пример Найти производную функции Это пример для самостоятельного решения (ответ в конце урока).
На практике правило дифференцирования сложной функции почти всегда применяется в комбинации с остальными правилами дифференцирования.
Пример Найти производную функции Сначала используем правило дифференцирования суммы, заодно в первом слагаемом выносим постоянный множитель за знак производной по правилу
:

В обоих слагаемых под штрихами у нас находится произведение функций, следовательно, нужно дважды применить правило
:
Замечаем, что под некоторыми штрихами у нас находятся сложные функции
,
. Каламбур, но это простейшие из сложных функций, и при определенном опыте решения производных Выбудете легко находить их устно.
А пока запишем подробно, согласно правилу
, получаем:
Готово.

  1   2   3   4   5   6   7   8

Как найти производную функции с примерами решения и готовыми ответами

Содержание:

  1. Определение производной
  2. Вычисление производной
  3. Основные правила дифференцирования
  4. Таблица производных основных элементарных и сложных функций
  5. Примеры с решением
  6. Пример 1.
  7. Пример 2.
  8. Пример 3.
  9. Пример 4.
  10. Определение производной функции, ее физический и геометрический смысл 
  11. Пример 5.
  12. Пример 6.
  13. Пример 7.

Определение производной

Предел отношения приращения функции к приращению аргумента, если приращение аргумента стремится к нулю и если этот предел существует, называется производной функции в точке (рис. 117).

.

Нахождение производной функции называется дифференцированием. Функция, дифференцируемая в каждой точке некоторого интервала, называется дифференцируемой на этом интервале.

Рис. 117

По этой ссылке вы найдёте полный курс лекций по высшей математике:

Вычисление производной

Основные правила дифференцирования

— постоянный множитель можно выносить за знак производной.

— производная суммы функций равна сумме производных этих функций.

Производная произведения двух функций вычисляется по формуле .

Производная отношения двух функций вычисляется по формуле .

Таблица производных основных элементарных и сложных функций

Уравнение касательной к графику функции в точке :

.

Возможно вам будут полезны данные страницы:

Примеры с решением

Пример 1.

Найдите производную функции

.

Решение:

Вынесем постоянный множитель за знак производной.

Применим формулу . Здесь .

.

Ответ: .

Пример 2.

Решите уравнение

.

Решение:

Область определения функции .

.

Уравнение имеет вид .

Ответ: 0.

Пример 3.

.

Решение:

Область определения функции: .

.

Уравнение имеет вид области определения

Ответ: 2.

Пример 4.

Решение:

Воспользуемся формулой

уравнение:

Ответ: -2.

Запомните!

Производная функции в точке равна угловому коэффициенту касательной, проведенной к графику функции в точке с абсциссой .

Угловой коэффициент прямой равен тангенсу угла, который эта прямая образует с положительным направлением оси абсцисс (рис. 118, 119).

В уравнении прямой линии число — угловой коэффициент.

Рис. 118 Рис. 119

Определение производной функции, ее физический и геометрический смысл 

Пусть дана функция . Фиксируем произвольную точку из области определения функции . Пусть — другая точка из , тогда разность называется приращением аргумента и обозначается а разность значений функции называется приращением функции и обозначается

Производной функции в точке называется предел отношения ее приращения в этой точке к приращению аргумента при (если этот предел существует). Производная обозначается или . Процесс нахождения производной называется дифференцированием. Итак, . В физическом смысле отношение является средней скоростью изменения функции на отрезке — Мгновенной скоростью изменения этой функции в точке .

Например, если функция задает зависимость пути , пройденного некоторым телом, от времени , то производная является скоростью движения; если функция дает зависимость количества производимой кем-то продукции от времени, то ее производная является производительностью труда.

Рис. 1

Геометрически отношение является тангенсом угла (рис. 1), который образует секущая, проходящая через точки и с осью . Если то точка стремится к точке , а угол стремится к углу который образует касательная к графику функции в точке с осью и, следовательно, . Отсюда получаем (геометрический смысл производной), где — угловой коэффициент касательной к графику функции в точке . Следовательно, уравнение касательной к графику функции в точке есть .

Функция называется дифференцируемой в точке , если ее приращение в этой точке можно представить в виде

(1)

где — константа, а — бесконечно малая функция (б. м.ф.) при .

Теорема 1. Необходимым и достаточным условием дифференцирования функции в некоторой точке является существование ее производной в этой точке.

Доказательство необходимости. Пусть функция дифференцируема в точке . Тогда , где — б.м.ф. при . . Отсюда по обратной лемме (см. п. 3, раздел 4.3) , т.е. производная функции существует.

Доказательство достаточности. Пусть существует производная функции . Согласно лемме (см. также п. 3 раздела 4,3) , где — б.м.ф. при , т.е. функция дифференцируема в точке .

Замечание. В ходе доказательства также получено, что для дифференцируемой функции равенство (1) можно записывать в виде , где — б.м.ф, при .

Теорема 2. Если функция дифференцируема в точке, то она непрерывна в ней.

Доказательство. Если функция дифференцируема в точке , то , где — б.м.ф. при . Откуда , т.е. функция непрерывна в точке .

Правила дифференцирования (нахождения производных функций). Производные основных элементарных функций:

  1. постоянная величина: ;
  2. степенная функция: ;
  3. показательная функция: ;
  4. показательная функция с основанием ;
  5. логарифмическая функция с натуральным логарифмом: ;
  6. логарифмическая функция: ;
  7. тригонометрические функции: .

Структурные правила дифференцирования:

  1. производная суммы: ;
  2. производная произведения: ;
  3. производная частного: ;
  4. производная сложной функции: ;
  5. производная обратной функции: . Вывод этих правил можно посмотреть в учебниках.

Производные неявных и параметрических функций. Пусть функция задана неявно при помощи уравнения . Чтобы найти её производную, нужно это равенство продифференцировать по с учетом зависимости от и затем решить полученное уравнение относительно .

Пример 5.

Найти , от неявной функции .

Решение:

Продифференцируем данное равенство по , затем найдем откуда .

Функция называется параметрической, если значения функции и значения аргумента задаются при помощи функций от вспомогательной величины , называемой параметром: и .

Правило нахождения производной параметрической функции: если существуют .

Пример 6.

Найти от функции , заданной параметрически: .

Решение:

, значит, .

Производные высших порядков. Производная функции является новой функцией того же аргумента . Производная этой функции называется производной 2-го порядка по отношению к исходной функции и она обозначается или . Аналогично определяются производная 3-го порядка, производная 4-го порядка и дальнейшие производные.

Пример 7.

Функция в точке 0 не имеет производной. Надо заменить ее параболой на некотором интервале, содержащем нуль, чтобы получившаяся функция всюду имела производную, как доказано на рис. 2. У Крылова в качестве функции было сопряжение круга с прямой.

Рис. 2

Решение:

Будем искать параболу в виде . Тогда в точке сопряжения параболы и правой половины графика выполнены уравнения (точка принадлежит обоим графикам и в этой точке равны производные):

Решая систему, получаем: , откуда искомая функция есть

Замечание.

В последние годы большую популярность в теории приближений в некоторых прикладных исследованиях приобрели сплайны.

Так называются функции, представленные на разных участках многочленами и имеющие непрерывные производные до какого-то порядка включительно. Крылов и мы, при решении задачи Крылова, как раз построили сплайн — составленную из параболы и двух прямых лучей функцию, имеющую непрерывную 1-ю производную (у Крылова сплайн имел непрерывную 2-ю производную).

§ 1.

Производная

Пусть значения переменных х и у связаны уравнением

F(xy) = 0.                                                                                                        (1)

Если функция = f(x), определенная на некотором интервале (а,в), такая, что уравнение (1) при подстановке в него вместо у выражения f(x) обращается в тождество, то говорят, что уравнение (1) задает функцию = f(x) неявно или что функция = f(x) есть неявная функция.

Укажем правило нахождения производной неявной функции, не преобразовывая ее в явную, то есть не представляя в виде = f(x), так как часто это преобразование бывает технически сложным или невозможным.

Для нахождения производной у’х неявной функции, нужно продифференцировать по х обе части равенства (1), учитывая, что у есть функция от х. Затем из полученного равенства выразить у’х.

Пример 1. Вычислить у’х.

У5+ху-х= 0

Продифференцируем обе части по х. Получим 5у4у’+у+ху’-2х=0Выразим у’. y(5у4) = 2х-у, у’ = (2х-у)/(5у4).

Пример 2.                                                                                                              

tg(x+yxy

Продифференцируем обе части по х. Получим  или . Отсюда или . Окончательно .

Заметим, что производная неявной функции выражается через х и у, то есть получается равенство

y‘ = g(xy)                                                                                                       (2)

Для вычисления второй производной неявной функции, нужно продифференцировать обе части равенства (2) по х и затем подставить выражение g(x, y) вместо y’. nx. Формулы производных высших порядков.

Производная экспоненты равна самой экспоненте (производная e в степени x равна e в степени x):
(1) (e x )′ = e x .

Производная показательной функции с основанием степени a равна самой функции, умноженной на натуральный логарифм от a :
(2) .

Вывод формулы производной экспоненты, e в степени x

Экспонента – это показательная функция, у которой основание степени равно числу e , которое является следующим пределом:
.
Здесь может быть как натуральным, так и действительным числом. Далее мы выводим формулу (1) производной экспоненты.

Вывод формулы производной экспоненты

Рассмотрим экспоненту, e в степени x :
y = e x .
Эта функция определена для всех . Найдем ее производную по переменной x . По определению, производная является следующим пределом:
(3) .

Преобразуем это выражение, чтобы свести его к известным математическим свойствам и правилам. Для этого нам понадобятся следующие факты:
А) Свойство экспоненты :
(4) ;
Б) Свойство логарифма :
(5) ;
В) Непрерывность логарифма и свойство пределов для непрерывной функции:
(6) .
Здесь – некоторая функция, у которой существует предел и этот предел положителен.
Г) Значение второго замечательного предела:
(7) .

Применяем эти факты к нашему пределу (3). Используем свойство (4):
;
.

Сделаем подстановку . Тогда ; .
В силу непрерывности экспоненты,
.
Поэтому при , . В результате получаем:
.

Сделаем подстановку . Тогда . При , . И мы имеем:
.

Применим свойство логарифма (5):
. Тогда
.

Применим свойство (6). Поскольку существует положительный предел и логарифм непрерывен, то:
.
Здесь мы также воспользовались вторым замечательным пределом (7). Тогда
.

Тем самым мы получили формулу (1) производной экспоненты.

Вывод формулы производной показательной функции

Теперь выведем формулу (2) производной показательной функции с основанием степени a . Мы считаем, что и . Тогда показательная функция
(8)
Определена для всех .

Преобразуем формулу (8). Для этого воспользуемся свойствами показательной функции и логарифма .
;
.
Итак, мы преобразовали формулу (8) к следующему виду:
.

Производные высших порядков от e в степени x

Теперь найдем производные высших порядков. Сначала рассмотрим экспоненту:
(14) .
(1) .

Мы видим, что производная от функции (14) равна самой функции (14). Дифференцируя (1), получаем производные второго и третьего порядка:
;
.

Отсюда видно, что производная n-го порядка также равна исходной функции:
.

Производные высших порядков показательной функции

Теперь рассмотрим показательную функцию с основанием степени a :
.
Мы нашли ее производную первого порядка:
(15) .

Дифференцируя (15), получаем производные второго и третьего порядка:
;
.

Мы видим, что каждое дифференцирование приводит к умножению исходной функции на . Поэтому производная n-го порядка имеет следующий вид:
.

На котором мы разобрали простейшие производные, а также познакомились с правилами дифференцирования и некоторыми техническими приемами нахождения производных. Таким образом, если с производными функций у Вас не очень или какие-нибудь моменты данной статьи будут не совсем понятны, то сначала ознакомьтесь с вышеуказанным уроком. Пожалуйста, настройтесь на серьезный лад – материал не из простых, но я все-таки постараюсь изложить его просто и доступно.

На практике с производной сложной функции приходится сталкиваться очень часто, я бы даже сказал, почти всегда, когда Вам даны задания на нахождение производных.

Смотрим в таблицу на правило (№5) дифференцирования сложной функции:

Разбираемся. Прежде всего, обратим внимание на запись . Здесь у нас две функции – и , причем функция , образно говоря, вложена в функцию . Функция такого вида (когда одна функция вложена в другую) и называется сложной функцией.

Функцию я буду называть внешней функцией , а функцию – внутренней (или вложенной) функцией .

! Данные определения не являются теоретическими и не должны фигурировать в чистовом оформлении заданий. Я применяю неформальные выражения «внешняя функция», «внутренняя» функция только для того, чтобы Вам легче было понять материал.

Для того, чтобы прояснить ситуацию, рассмотрим:

Пример 1

Найти производную функции

Под синусом у нас находится не просто буква «икс», а целое выражение , поэтому найти производную сразу по таблице не получится. Также мы замечаем, что здесь невозможно применить первые четыре правила, вроде бы есть разность, но дело в том, что «разрывать на части» синус нельзя:

В данном примере уже из моих объяснений интуитивно понятно, что функция – это сложная функция, причем многочлен является внутренней функцией (вложением), а – внешней функцией.

Первый шаг , который нужно выполнить при нахождении производной сложной функции состоит в том, чтобы разобраться, какая функция является внутренней, а какая – внешней .

В случае простых примеров вроде понятно, что под синус вложен многочлен . А как же быть, если всё не очевидно? Как точно определить, какая функция является внешней, а какая внутренней? Для этого я предлагаю использовать следующий прием, который можно проводить мысленно или на черновике.

Представим, что нам нужно вычислить на калькуляторе значение выражения при (вместо единицы может быть любое число).

Что мы вычислим в первую очередь? В первую очередь нужно будет выполнить следующее действие: , поэтому многочлен и будет внутренней функцией :

Во вторую очередь нужно будет найти , поэтому синус – будет внешней функцией:

После того, как мы РАЗОБРАЛИСЬ с внутренней и внешней функциями самое время применить правило дифференцирования сложной функции .

Начинаем решать. Из урока Как найти производную? мы помним, что оформление решения любой производной всегда начинается так – заключаем выражение в скобки и ставим справа вверху штрих:

Сначала находим производную внешней функции (синуса), смотрим на таблицу производных элементарных функций и замечаем, что . Все табличные формулы применимы и в том, случае, если «икс» заменить сложным выражением , в данном случае:

Обратите внимание, что внутренняя функция не изменилась, её мы не трогаем .

Ну и совершенно очевидно, что

Результат применения формулы в чистовом оформлении выглядит так:

Постоянный множитель обычно выносят в начало выражения:

Если осталось какое-либо недопонимание, перепишите решение на бумагу и еще раз прочитайте объяснения.

Пример 2

Найти производную функции

Пример 3

Найти производную функции

Как всегда записываем:

Разбираемся, где у нас внешняя функция, а где внутренняя. Для этого пробуем (мысленно или на черновике) вычислить значение выражения при . Что нужно выполнить в первую очередь? В первую очередь нужно сосчитать чему равно основание: , значит, многочлен – и есть внутренняя функция:

И, только потом выполняется возведение в степень , следовательно, степенная функция – это внешняя функция:

Согласно формуле , сначала нужно найти производную от внешней функции, в данном случае, от степени. Разыскиваем в таблице нужную формулу: . Повторяем еще раз: любая табличная формула справедлива не только для «икс», но и для сложного выражения . Таким образом, результат применения правила дифференцирования сложной функции следующий:

Снова подчеркиваю, что когда мы берем производную от внешней функции , внутренняя функция у нас не меняется:

Теперь осталось найти совсем простую производную от внутренней функции и немного «причесать» результат:

Пример 4

Найти производную функции

Это пример для самостоятельного решения (ответ в конце урока).

Для закрепления понимания производной сложной функции приведу пример без комментариев, попробуйте самостоятельно разобраться, порассуждать, где внешняя и где внутренняя функция, почему задания решены именно так?

Пример 5

а) Найти производную функции

б) Найти производную функции

Пример 6

Найти производную функции

Здесь у нас корень, а для того, чтобы продифференцировать корень, его нужно представить в виде степени . Таким образом, сначала приводим функцию в надлежащий для дифференцирования вид:

Анализируя функцию, приходим к выводу, что сумма трех слагаемых – это внутренняя функция, а возведение в степень – внешняя функция. Применяем правило дифференцирования сложной функции :

Степень снова представляем в виде радикала (корня), а для производной внутренней функции применяем простое правило дифференцирования суммы:

Готово. Можно еще в скобках привести выражение к общему знаменателю и записать всё одной дробью. Красиво, конечно, но когда получаются громоздкие длинные производные – лучше этого не делать (легко запутаться, допустить ненужную ошибку, да и преподавателю будет неудобно проверять).

Пример 7

Найти производную функции

Это пример для самостоятельного решения (ответ в конце урока).

Интересно отметить, что иногда вместо правила дифференцирования сложной функции можно использовать правило дифференцирования частного , но такое решение будет выглядеть как извращение необычно. Вот характерный пример:

Пример 8

Найти производную функции

Здесь можно использовать правило дифференцирования частного , но гораздо выгоднее найти производную через правило дифференцирования сложной функции:

Подготавливаем функцию для дифференцирования – выносим минус за знак производной, а косинус поднимаем в числитель:

Косинус – внутренняя функция, возведение в степень – внешняя функция.
Используем наше правило :

Находим производную внутренней функции, косинус сбрасываем обратно вниз:

Готово. В рассмотренном примере важно не запутаться в знаках. Кстати, попробуйте решить его с помощью правила , ответы должны совпасть.

Пример 9

Найти производную функции

Это пример для самостоятельного решения (ответ в конце урока).

До сих пор мы рассматривали случаи, когда у нас в сложной функции было только одно вложение. В практических же заданиях часто можно встретить производные, где, как матрешки, одна в другую, вложены сразу 3, а то и 4-5 функций.

Пример 10

Найти производную функции

Разбираемся во вложениях этой функции. Пробуем вычислить выражение с помощью подопытного значения . Как бы мы считали на калькуляторе?

Сначала нужно найти , значит, арксинус – самое глубокое вложение:

Затем этот арксинус единицы следует возвести в квадрат :

И, наконец, семерку возводим в степень :

То есть, в данном примере у нас три разные функции и два вложения, при этом, самой внутренней функцией является арксинус, а самой внешней функцией – показательная функция.

Начинаем решать

Согласно правилу сначала нужно взять производную от внешней функции. Смотрим в таблицу производных и находим производную показательной функции: Единственное отличие – вместо «икс» у нас сложное выражение , что не отменяет справедливость данной формулы. Итак, результат применения правила дифференцирования сложной функции следующий.

Операция отыскания производной называется дифференцированием.

В результате решения задач об отыскании производных у самых простых (и не очень простых) функций по определению производной как предела отношения приращения к приращению аргумента появились таблица производных и точно определённые правила дифференцирования. Первыми на ниве нахождения производных потрудились Исаак Ньютон (1643-1727) и Готфрид Вильгельм Лейбниц (1646-1716).

Поэтому в наше время, чтобы найти производную любой функции, не надо вычислять упомянутый выше предел отношения приращения функции к приращению аргумента, а нужно лишь воспользоваться таблицей производных и правилами дифференцирования. Для нахождения производной подходит следующий алгоритм.

Чтобы найти производную , надо выражение под знаком штриха разобрать на составляющие простые функции и определить, какими действиями (произведение, сумма, частное) связаны эти функции. Далее производные элементарных функций находим в таблице производных, а формулы производных произведения, суммы и частного – в правилах дифференцирования. Таблица производных и правила дифференцирования даны после первых двух примеров.

Пример 1. Найти производную функции

Решение. Из правил дифференцирования выясняем, что производная суммы функций есть сумма производных функций, т. е.

Из таблицы производных выясняем, что производная “икса” равна единице, а производная синуса – косинусу. Подставляем эти значения в сумму производных и находим требуемую условием задачи производную:

Пример 2. Найти производную функции

Решение. Дифференцируем как производную суммы, в которой второе слагаемое с постоянным множителем, его можно вынести за знак производной:

Если пока возникают вопросы, откуда что берётся, они, как правило, проясняются после ознакомления с таблицей производных и простейшими правилами дифференцирования. К ним мы и переходим прямо сейчас.

Таблица производных простых функций

Правила дифференцирования

1. Производная суммы или разности
2. Производная произведения
2a. Производная выражения, умноженного на постоянный множитель
3. Производная частного
4. Производная сложной функции

Правило 1. Если функции

дифференцируемы в некоторой точке , то в той же точке дифференцируемы и функции

причём

т.е. производная алгебраической суммы функций равна алгебраической сумме производных этих функций.

Следствие. Если две дифференцируемые функции отличаются на постоянное слагаемое, то их производные равны , т.е.

Правило 2. Если функции

дифференцируемы в некоторой точке , то в то же точке дифференцируемо и их произведение

причём

т. е. производная произведения двух функций равна сумме произведений каждой из этих функций на производную другой.

Следствие 1. Постоянный множитель можно выносить за знак производной :

Следствие 2. Производная произведения нескольких дифференцируемых функций равна сумме произведений производной каждого из сомножителей на все остальные.

Например, для трёх множителей:

Правило 3. Если функции

дифференцируемы в некоторой точке и , то в этой точке дифференцируемо и их частное u/v , причём

т.е. производная частного двух функций равна дроби, числитель которой есть разность произведений знаменателя на производную числителя и числителя на производную знаменателя, а знаменатель есть квадрат прежнего числителя.

Где что искать на других страницах

При нахождении производной произведения и частного в реальных задачах всегда требуется применять сразу несколько правил дифференцирования, поэтому больше примеров на эти производные – в статье “Производная произведения и частного функций ” .

Замечание. Следует не путать константу (то есть, число) как слагаемое в сумме и как постоянный множитель! В случае слагаемого её производная равна нулю, а в случае постоянного множителя она выносится за знак производных. Это типичная ошибка, которая встречается на начальном этапе изучения производных, но по мере решения уже нескольких одно- двухсоставных примеров средний студент этой ошибки уже не делает.

А если при дифференцировании произведения или частного у вас появилось слагаемое u v , в котором u – число, например, 2 или 5, то есть константа, то производная этого числа будет равна нулю и, следовательно, всё слагаемое будет равно нулю (такой случай разобран в примере 10).

Другая частая ошибка – механическое решение производной сложной функции как производной простой функции. Поэтому производной сложной функции посвящена отдельная статья. Но сначала будем учиться находить производные простых функций.

По ходу не обойтись без преобразований выражений. Для этого может потребоваться открыть в новых окнах пособия Действия со степенями и корнями и Действия с дробями .

Если Вы ищете решения производных дробей со степенями и корнями, то есть, когда функция имеет вид вроде , то следуйте на занятие “Производная суммы дробей со степенями и корнями “.

Если же перед Вами задача вроде , то Вам на занятие “Производные простых тригонометрических функций”.

Пошаговые примеры – как найти производную

Пример 3. Найти производную функции

Решение. Определяем части выражения функции: всё выражение представляет произведение, а его сомножители – суммы, во второй из которых одно из слагаемых содержит постоянный множитель. Применяем правило дифференцирования произведения: производная произведения двух функций равна сумме произведений каждой из этих функций на производную другой:

Далее применяем правило дифференцирования суммы: производная алгебраической суммы функций равна алгебраической сумме производных этих функций. В нашем случае в каждой сумме второе слагаемое со знаком минус. В каждой сумме видим и независимую переменную, производная которой равна единице, и константу (число), производная которой равна нулю. Итак, “икс” у нас превращается в единицу, а минус 5 – в ноль. Во втором выражении “икс” умножен на 2, так что двойку умножаем на ту же единицу как производную “икса”. Получаем следующие значения производных:

Подставляем найденные производные в сумму произведений и получаем требуемую условием задачи производную всей функции:

Пример 4. Найти производную функции

Решение. От нас требуется найти производную частного. Применяем формулу дифференцирования частного: производная частного двух функций равна дроби, числитель которой есть разность произведений знаменателя на производную числителя и числителя на производную знаменателя, а знаменатель есть квадрат прежнего числителя. Получаем:

Производную сомножителей в числителе мы уже нашли в примере 2. Не забудем также, что произведение, являющееся вторым сомножителем в числителе в текущем примере берётся со знаком минус:

Если Вы ищете решения таких задач, в которых надо найти производную функции, где сплошное нагромождение корней и степеней, как, например, , то добро пожаловать на занятие “Производная суммы дробей со степенями и корнями” .

Если же Вам нужно узнать больше о производных синусов, косинусов, тангенсов и других тригонометрических функций, то есть, когда функция имеет вид вроде , то Вам на урок “Производные простых тригонометрических функций” .

Пример 5. Найти производную функции

Решение. В данной функции видим произведение, один из сомножителей которых – квадратный корень из независимой переменной, с производной которого мы ознакомились в таблице производных. По правилу дифференцирования произведения и табличному значению производной квадратного корня получаем:

Пример 6. Найти производную функции

Решение. В данной функции видим частное, делимое которого – квадратный корень из независимой переменной. По правилу дифференцирования частного, которое мы повторили и применили в примере 4, и табличному значению производной квадратного корня получаем:

Чтобы избавиться от дроби в числителе, умножаем числитель и знаменатель на .

Вычисление производной – одна из самых важных операций в дифференциальном исчислении. Ниже приводится таблица нахождения производных простых функций. Более сложные правила дифференцирования смотрите в других уроках:
  • Таблица производных экспоненциальных и логарифмических функций
Приведенные формулы используйте как справочные значения. Они помогут в решении дифференциальных уравнений и задач. На картинке, в таблице производных простых функций, приведена “шпаргалка” основных случаев нахождения производной в понятном для применения виде, рядом с ним даны пояснения для каждого случая.

Производные простых функций

1. Производная от числа равна нулю
с´ = 0
Пример:
5´ = 0

Пояснение :
Производная показывает скорость изменения значения функции при изменении аргумента. Поскольку число никак не меняется ни при каких условиях – скорость его изменения всегда равна нулю.

2. Производная переменной равна единице
x´ = 1

Пояснение :
При каждом приращении аргумента (х) на единицу значение функции (результата вычислений) увеличивается на эту же самую величину. Таким образом, скорость изменения значения функции y = x точно равна скорости изменения значения аргумента.

3. Производная переменной и множителя равна этому множителю
сx´ = с
Пример:
(3x)´ = 3
(2x)´ = 2
Пояснение :
В данном случае, при каждом изменении аргумента функции (х ) ее значение (y) растет в с раз. Таким образом, скорость изменения значения функции по отношению к скорости изменения аргумента точно равно величине с .

Откуда следует, что
(cx + b)” = c
то есть дифференциал линейной функции y=kx+b равен угловому коэффициенту наклона прямой (k).


4. Производная переменной по модулю равна частному этой переменной к ее модулю
|x|” = x / |x| при условии, что х ≠ 0
Пояснение :
Поскольку производная переменной (см. формулу 2) равна единице, то производная модуля отличается лишь тем, что значение скорости изменения функции меняется на противоположное при пересечении точки начала координат (попробуйте нарисовать график функции y = |x| и убедитесь в этом сами. Именно такое значение и возвращает выражение x / |x| . Когда x 0 – единице. То есть при отрицательных значениях переменной х при каждом увеличении изменении аргумента значение функции уменьшается на точно такое же значение, а при положительных – наоборот, возрастает, но точно на такое же значение.

5. Производная переменной в степени равна произведению числа этой степени и переменной в степени, уменьшенной на единицу
(x c)”= cx c-1 , при условии, что x c и сx c-1 ,определены а с ≠ 0
Пример:
(x 2)” = 2x
(x 3)” = 3x 2
Для запоминания формулы :
Снесите степень переменной “вниз” как множитель, а потом уменьшите саму степень на единицу. Например, для x 2 – двойка оказалась впереди икса, а потом уменьшенная степень (2-1=1) просто дала нам 2х. То же самое произошло для x 3 – тройку “спускаем вниз”, уменьшаем ее на единицу и вместо куба имеем квадрат, то есть 3x 2 . Немного “не научно”, но очень просто запомнить.

6. Производная дроби 1/х
(1/х)” = – 1 / x 2
Пример:
Поскольку дробь можно представить как возведение в отрицательную степень
(1/x)” = (x -1)” , тогда можно применить формулу из правила 5 таблицы производных
(x -1)” = -1x -2 = – 1 / х 2

7. Производная дроби с переменной произвольной степени в знаменателе
(1 / x c)” = – c / x c+1
Пример:
(1 / x 2)” = – 2 / x 3

8. Производная корня (производная переменной под квадратным корнем)
(√x)” = 1 / (2√x) или 1/2 х -1/2
Пример:
(√x)” = (х 1/2)” значит можно применить формулу из правила 5
(х 1/2)” = 1/2 х -1/2 = 1 / (2√х)

9. Производная переменной под корнем произвольной степени
(n √x)” = 1 / (n n √x n-1)

При выводе самой первой формулы таблицы будем исходить из определения производнойфункции в точке. Возьмем , где x – любое действительное число, то есть, x – любое число из области определения функции . Запишем предел отношения приращения функции к приращению аргумента при :

Следует заметить, что под знаком предела получается выражение , которое не являетсянеопределенностью ноль делить на ноль, так как в числителе находится не бесконечно малая величина, а именно ноль. Другими словами, приращение постоянной функции всегда равно нулю.

Таким образом, производная постоянной функции равна нулю на всей области определения .

Производная степенной функции.

Формула производной степенной функции имеет вид , где показатель степени p – любое действительное число.

Докажем сначала формулу для натурального показателя степени, то есть, для p = 1, 2, 3, …

Будем пользоваться определением производной. Запишем предел отношения приращения степенной функции к приращению аргумента:

Для упрощения выражения в числителе обратимся к формуле бинома Ньютона:

Следовательно,

Этим доказана формула производной степенной функции для натурального показателя.

Производная показательной функции.

Вывод формулы производной приведем на основе определения:

Пришли к неопределенности. Для ее раскрытия введем новую переменную , причем при . Тогда . В последнем переходе мы использовали формулу перехода к новому основанию логарифма.

Выполним подстановку в исходный предел:

Если вспомнить второй замечательный предел, то придем к формуле производной показательной функции:

Производная логарифмической функции.

Докажем формулу производной логарифмической функции для всех x из области определения и всех допустимых значениях основания a логарифма. По определению производной имеем:

Как Вы заметили, при доказательстве преобразования проводились с использованием свойств логарифма. Равенство справедливо в силу второго замечательного предела.

Производные тригонометрических функций.

Для вывода формул производных тригонометрических функций нам придется вспомнить некоторые формулы тригонометрии, а также первый замечательный предел.

По определению производной для функции синуса имеем .

Воспользуемся формулой разности синусов:

Осталось обратиться к первому замечательному пределу:

Таким образом, производная функции sin x есть cos x .

Абсолютно аналогично доказывается формула производной косинуса.

Следовательно, производная функции cos x есть –sin x .

Вывод формул таблицы производных для тангенса и котангенса проведем с использованием доказанных правил дифференцирования (производная дроби).

Производные гиперболических функций.

Правила дифференцирования и формула производной показательной функции из таблицы производных позволяют вывести формулы производных гиперболического синуса, косинуса, тангенса и котангенса.

Производная обратной функции.

Чтобы при изложении не было путаницы, давайте обозначать в нижнем индексе аргумент функции, по которому выполняется дифференцирование, то есть, – это производная функции f(x) по x .

Теперь сформулируем правило нахождения производной обратной функции.

Пусть функции y = f(x) и x = g(y) взаимно обратные, определенные на интервалах и соответственно. Если в точке существует конечная отличная от нуля производная функции f(x) , то в точке существует конечная производная обратной функции g(y) , причем . В другой записи .

Можно это правило переформулировать для любого x из промежутка , тогда получим .

Давайте проверим справедливость этих формул.

Найдем обратную функцию для натурального логарифма (здесь y – функция, а x – аргумент). Разрешив это уравнение относительно x , получим (здесь x – функция, а y – ее аргумент). То есть, и взаимно обратные функции.

Из таблицы производных видим, что и .

Убедимся, что формулы нахождения производных обратной функции приводят нас к этим же результатам:

10.1 Производные комплексных функций

10.1 Производные комплексных функций
Далее: 10.2 Дифференцируемые функции на Up: 10. Производная Предыдущая: 10. Производная & nbsp Индекс

Вы знакомы с производными функций от к , и с Мотивация определения производной как наклона касательной к кривой. Для сложных функций геометрическая мотивация отсутствует, но определение формально то же, что и определение производных действительных функций.

По определению предела можно сказать, что дифференцируемо в если , и является предельной точкой и существует функция такой которая непрерывна при, и такой, что

(10.2)

и в этом случае равно.

Иногда полезно перефразировать условие (10.2) следующим образом: является дифференцируемый в, если , является предельной точкой , и есть функция такая, что непрерывна в точке, и

(10.3)

В таком случае, . 10,4 Замечание. Непосредственно из (10.3) следует, что если дифференцируема в точке, тогда непрерывна при.

Доказательство. Поскольку в точке дифференцируемы, существуют функции , такой, что, непрерывны при, а




Следует, что

и непрерывна при.

Мы можем позволить и мы видим дифференцируемо при а также

Доказательство: доказательство предоставляется вам.

Доказательство: согласно нашим предположениям, существуют функции


такая, что непрерывна в точке, непрерывна в точке а также

Если , тогда , поэтому мы можем заменить в (10. 15) с помощью, чтобы получить

Используя (10.14) для переписывания, получаем

Следовательно, мы имеем

и непрерывна при. Следовательно, дифференцируемо в и

Доказательство: если , мы видели выше, что это дифференцируемый и . Позвольте быть комплексной функцией, и позволять .Предположим, дифференцируем в, и. потом . По цепному правилу дифференцируем в, и




Далее: 10.2 Дифференцируемые функции на Up: 10. Производная Предыдущая: 10. Производная & nbsp Индекс

3.3 Правила дифференциации – Объем исчисления 1

Цели обучения

  • Укажите правила постоянной, постоянной кратной и степени.
  • Примените правила суммы и разницы для комбинирования производных финансовых инструментов.
  • Используйте правило произведения, чтобы найти производную произведения функций.
  • Используйте правило частного для нахождения производной частного от функций.
  • Распространить правило мощности на функции с отрицательными показателями.
  • Комбинируйте правила дифференцирования, чтобы найти производную полиномиальной или рациональной функции.

Нахождение производных функций с использованием определения производной может быть длительным и, для некоторых функций, довольно сложным процессом.Например, ранее мы обнаружили это с помощью процесса, который включал умножение выражения на конъюгат перед вычислением предела. Процесс, который мы могли бы использовать для оценки с использованием определения, хотя и похож, но более сложен. В этом разделе мы разрабатываем правила для поиска производных, которые позволяют нам обойти этот процесс. Начнем с основ.

Функции и где положительное целое число являются строительными блоками, из которых строятся все полиномы и рациональные функции.Чтобы найти производные многочленов и рациональных функций эффективно, не прибегая к предельному определению производной, мы должны сначала разработать формулы для дифференцирования этих основных функций.

Мы показали, что

а также .

На этом этапе вы можете увидеть, как начинает развиваться паттерн для производных формы. Мы продолжим рассмотрение производных формул, дифференцируя степенные функции вида где – натуральное число. Формулы для производных этого типа функций мы разрабатываем поэтапно, начиная с положительных целых степеней.Прежде чем сформулировать и доказать общее правило для производных функций этого вида, рассмотрим конкретный случай,.

Дифференциация

Найти.

Решение

Найти.

Решение

Как мы увидим, процедура нахождения производной общего вида очень похожа. Хотя часто неразумно делать общие выводы из конкретных примеров, мы отмечаем, что, когда мы проводим дифференциацию, степень включения становится коэффициентом в производной, а степень включения в производной уменьшается на 1.Следующая теорема утверждает, что это правило степени выполняется для всех положительных целых степеней. В конечном итоге мы расширим этот результат до отрицательных целых степеней. Позже мы увидим, что это правило может быть распространено сначала на рациональные степени, а затем на произвольные степени. Однако имейте в виду, что это правило не применяется к функциям, в которых константа возводится в переменную степень, например.

Правило силы

Позвольте быть положительное целое число. Если, то

.

В качестве альтернативы мы можем выразить это правило как

.

Мы находим наши следующие правила дифференцирования, рассматривая производные сумм, разностей и постоянных кратных функций. Как и при работе с функциями, существуют правила, которые упрощают поиск производных функций, которые мы складываем, вычитаем или умножаем на константу. Эти правила кратко изложены в следующей теореме.

Теперь, когда мы изучили основные правила, мы можем приступить к рассмотрению некоторых из более сложных правил.Первый исследует производную от произведения двух функций. Хотя может возникнуть соблазн предположить, что производная продукта является произведением производных инструментов, подобно правилам суммы и разности, правило продукта не следует этому шаблону. Чтобы понять, почему мы не можем использовать этот шаблон, рассмотрим функцию, у которой производная есть, а нет.

Проба

Начнем с предположения, что и являются дифференцируемыми функциями. В ключевой точке этого доказательства нам нужно использовать тот факт, что, поскольку оно дифференцируемо, оно также непрерывно.В частности, мы используем тот факт, что поскольку непрерывно,.

Применяя определение предела производной к, получаем

.

Добавляя и вычитая в числителе, получаем

.

После разделения этого частного и применения закона сумм для пределов производная становится

.

Переставляя, получаем

.

Используя непрерывность определения производных от и и применяя предельные законы, мы приходим к правилу произведения,

Применение правила произведения к постоянным функциям

Применение правила произведения к биномам

Для, найдите, применив правило произведения.Проверьте результат, сначала найдя продукт, а затем дифференцируя его.

Используйте правило произведения, чтобы получить производную от.

Решение

.

Разработав и применив правило продукта, мы теперь рассмотрим дифференцирование частных функций. Как мы видим в следующей теореме, производная частного не является частным производных; скорее, это производная функции в числителе, умноженная на функцию в знаменателе, минус производная функции в знаменателе, умноженная на функцию в числителе, деленную на квадрат функции в знаменателе.Чтобы лучше понять, почему мы не можем просто взять частное производных, имейте в виду, что

, что не то же самое, что.

Доказательство правила частного очень похоже на доказательство правила произведения, поэтому здесь оно опускается. Вместо этого мы применяем это новое правило для поиска производных в следующем примере.

Применение правила частного

Используйте правило частного, чтобы найти производную от.

Найдите производную от.

Решение

.

Теперь можно использовать правило частного, чтобы расширить правило мощности для нахождения производных функций вида где – отрицательное целое число.

Правило расширенной мощности

Если – отрицательное целое число, то

.

Проба

Если – отрицательное целое число, мы можем установить, так что это положительное целое число с. Поскольку для каждого положительного целого числа мы можем теперь применить правило частного, установив и. В этом случае и. Таким образом,

.

Упрощая, мы видим, что

.

Наконец, заметьте, что, поскольку при замене мы получаем

Использование правила расширенной мощности

Найти.

Решение

Применяя расширенное правило мощности с, получаем

.

Использование правила расширенной мощности и правила постоянной множественности

Для нахождения используйте расширенное правило мощности и правило множественности констант.

Решение

Может показаться заманчивым использовать правило частного для нахождения этой производной, и, конечно, это было бы правильно.Однако гораздо проще отличить эту функцию, сначала переписав ее как.

Найдите производную от использования расширенного правила мощности.

Решение

.

Как мы видели в примерах в этом разделе, редко бывает, что нам приходится применять только одно правило дифференцирования, чтобы найти производную заданной функции. На этом этапе, комбинируя правила дифференцирования, мы можем найти производные любой полиномиальной или рациональной функции.Позже мы столкнемся с более сложными комбинациями правил дифференцирования. Хорошее практическое правило, которое следует использовать при применении нескольких правил, – применять правила в обратном порядке, в котором мы будем оценивать функцию.

Объединение правил дифференциации

Для, найдите.

Решение

Для поиска этой производной требуется правило сумм, правило множественных постоянных и правило произведения.

Расширение правила продукта

For, выразить через и их производные.

Объединение правила частного и правила произведения

Для, найдите.

Решение

Эта процедура типична для нахождения производной рациональной функции.

Найти.

Решение

.

Определение того, где функция имеет тангенс по горизонтали

Определите значения, для которых имеется горизонтальная касательная.

Найдите значение (я), для которого прямая, касательная к графику, параллельна прямой.

Решение

Студенческий проект – Трибуны Формулы-1

Автомобильные гонки

Formula One могут быть очень захватывающими и привлечь внимание множества зрителей. Дизайнеры трассы Формулы-1 должны обеспечить наличие достаточного пространства на трибунах вокруг трассы для размещения этих зрителей. Однако автомобильные гонки могут быть опасными, и соображения безопасности имеют первостепенное значение. Трибуны должны быть размещены там, где зрителям не будет угрожать опасность, если водитель потеряет управление автомобилем ((Рисунок)).

Рисунок 1. Трибуна рядом с выездом на гоночную трассу Circuit de Barcelona-Catalunya, где зрителям ничего не угрожает.

**********

Безопасность особенно важна на поворотах. Если водитель не замедлит достаточную скорость перед входом в поворот, автомобиль может соскользнуть с гоночной трассы. Обычно это приводит к более широкому повороту, что замедляет водителя. Но если водитель полностью потеряет контроль, автомобиль может полностью вылететь за пределы трассы по траектории, касательной к изгибу гоночной трассы.

Предположим, вы разрабатываете новую трассу Формулы-1. Один участок пути можно смоделировать функцией ((Рисунок)). Текущий план требует, чтобы трибуны были построены вдоль первой прямой и вокруг части первой кривой. Планируется, что передний угол трибуны должен быть расположен в этой точке. Мы хотим определить, угрожает ли это место зрителям, если водитель потеряет контроль над автомобилем.

Рис. 2. (a) Одна часть ипподрома может быть смоделирована функцией.(б) Передний угол трибуны расположен по адресу.
  1. Физики определили, что водители, скорее всего, потеряют контроль над своими автомобилями при входе в поворот в точке, где наклон касательной линии равен 1. Найдите координаты этой точки рядом с поворотом.
  2. Найдите уравнение касательной к кривой в этой точке.
  3. Чтобы определить, находятся ли зрители в опасности в этом сценарии, найдите -координату точки, в которой касательная линия пересекает линию.Безопасно ли эта точка справа от трибуны? Или зрители в опасности?
  4. Что делать, если водитель потеряет управление раньше, чем проект физиков? Предположим, водитель теряет контроль в точке. Каков наклон касательной в этой точке?
  5. Если водитель теряет управление, как описано в части 4, безопасны ли зрители?
  6. Следует продолжить текущий дизайн трибуны или трибуны следует переместить?

Найдите для каждой функции следующие упражнения.

1.

2.

Решение

3.

4.

Решение

5.

6.

Решение

7.

8.

Решение

9.

10.

Решение

11.

12.

Решение

Для следующих упражнений найдите уравнение касательной к графику данной функции в указанной точке. Воспользуйтесь графическим калькулятором, чтобы построить график функции и касательной.

13. [T] at

15. [T] at

Для следующих упражнений предположим, что обе функции и являются дифференцируемыми для всех.Найдите производную каждой функции.

17.

18.

Решение

19.

20.

Решение

Для следующих упражнений предположим, что обе и являются дифференцируемыми функциями со значениями, указанными в следующей таблице. Используйте следующую таблицу для расчета следующих производных.

21. Найдите, если.

23. Найдите, если.

24. Найдите, если.

Для следующих упражнений используйте следующий рисунок, чтобы найти указанные производные, если они существуют.

Решение

а. 2
г. не существует
c. 2,5

Для следующих упражнений

  1. Оценить и
  2. Постройте график функции и касательной в точке.

28.[Т]

Решение

а. 23
г.

29. [Т]

30. [Т]

Решение

а. 3
г.

31. [Т]

32. Найдите уравнение касательной к графику at.

Решение

33. Найдите уравнение касательной к графику при.

34. Найдите уравнение касательной к графику at.

Решение

35. Найдите на графике точку, в которой касательная линия в этой точке имеет точку пересечения 6.

36. Найдите уравнение прямой, проходящей через точку и касательную к графику.

Решение

37. Определите все точки на графике, для которых наклон касательной равен

  1. горизонтальный
  2. -1.

38. Найдите квадратный многочлен такой, что, и.

Решение

39. Автомобиль, едущий по скоростной автомагистрали, проехал метры за секунды.

  1. Определите время в секундах, когда скорость автомобиля равна 0.
  2. Определите ускорение автомобиля при скорости 0.

40. [T] Селедка, плывущая по прямой, прошла ноги за секунды.

Определите скорость сельди, пройдя 3 секунды.

Решение

или 0,0992 фут / с

41. Численность миллионов камбал в Атлантическом океане моделируется функцией, где измеряется в годах.

  1. Определите начальную популяцию камбалы.
  2. Определите и кратко интерпретируйте результат.
Решение

а.
г. -0,02395 мг / л-час, -0.01344 мг / л-час, -0,003566 мг / л-час, -0,001579 мг / л-час
c. Скорость, с которой снижается концентрация лекарственного средства в кровотоке, замедляется до 0 с увеличением времени.

43. У книжного издателя есть функция затрат, определяемая выражением, где – количество копий книги в тысячах, а – стоимость одной книги, измеренная в долларах. Оцените и объясните его значение.

Решение

а.
г. Н / м

2.6: Уравнения Коши-Римана – Математика LibreTexts

Уравнения Коши-Римана являются нашим первым следствием того факта, что предел, определяющий \ (f (z) \), должен быть одинаковым независимо от того, в каком направлении вы приближаетесь к \ (z \ ) из.Уравнения Коши-Римана станут одним из самых важных инструментов в нашем наборе инструментов.

2.7.1 Частные производные как лимиты

Прежде чем перейти к уравнениям Коши-Римана, напомним о частных производных. Если \ (u (x, y) \) является функцией двух переменных, то частные производные от \ (u \) определяются как

\ [\ dfrac {\ partial u} {\ partial x} (x, y) = \ lim _ {\ Delta x \ to 0} \ dfrac {u (x + \ Delta, y) – u (x, y) } {\ Delta x}, \]

, т. Е. Производная от \ (u \), сохраняющая постоянную \ (y \).

\ [\ dfrac {\ partial u} {\ partial y} (x, y) = \ lim _ {\ Delta y \ to 0} \ dfrac {u (x, y + \ Delta y) – u (x, y )} {\ Delta y}, \]

, т. Е. Производная от \ (u \), сохраняющая постоянную \ (x \).

2.7.2 Уравнения Коши-Римана

Уравнения Коши-Римана используют частные производные от \ (u \) и \ (v \), чтобы позволить нам делать две вещи: во-первых, проверять, имеет ли \ (f \) комплексную производную, и, во-вторых, вычислять, что производная. Начнем с формулировки уравнений в виде теоремы.

Теорема \ (\ PageIndex {1} \): уравнения Коши-Римана

Если \ (f (z) = u (x, y) + iv (x, y) \) аналитический (комплексно дифференцируемый), то

\ [f ‘(z) = \ dfrac {\ partial u} {\ partial x} + i \ dfrac {\ partial v} {\ partial x} = \ dfrac {\ partial v} {\ partial y} – i \ dfrac {\ partial u} {\ partial y} \]

В частности,

\ [\ dfrac {\ partial u} {\ partial x} = \ dfrac {\ partial v} {\ partial y} \ text {и} \ dfrac {\ partial u} {\ partial y} = – \ dfrac { \ partial v} {\ partial x}.\]

Эту последнюю систему дифференциальных уравнений в частных производных обычно понимают под уравнениями Коши-Римана.

Вот краткая форма уравнений Коши-Римана:

\ [u_x = v_y \]

\ [u_y = -v_x \]

Проба

Предположим, что \ (f (z) \) дифференцируема в некоторой области \ (A \) и

\ [f (z) = f (x + iy) = u (x, y) + iv (x, y). \]

Мы вычислим \ (f ‘(z) \), приближаясь к \ (z \) сначала в горизонтальном направлении, а затем в вертикальном направлении.Мы будем использовать формулу

\ [f ‘(z) = \ lim _ {\ Delta \ to 0} \ dfrac {f (z + \ Delta z) – f (z)} {\ Delta z}, \]

, где \ (\ Delta z = \ Delta x + i \ Delta y \).

Горизонтальное направление: \ (\ Delta y = 0, \ Delta z = \ Delta x \)

\ [\ begin {array} {rcl} {f ‘(z)} & = & {\ lim _ {\ Delta z \ to 0} \ dfrac {f (z + \ Delta z) – f (z)} { \ Delta z}} \\ {} & = & {\ lim _ {\ Delta x \ to 0} \ dfrac {f (x + \ Delta x + iy) – f (x + iy)} {\ Delta x}} \\ {} & = & {\ lim _ {\ Delta x \ to 0} \ dfrac {(u (x + \ Delta, y) + iv (x + \ Delta x, y)) – (u (x, y ) + iv (x, y))} {\ Delta x}} \\ {} & = & {\ lim _ {\ Delta x \ to 0} \ dfrac {u (x + \ Delta x, y) – u ( x, y)} {\ Delta x} + i \ dfrac {v (x + \ Delta x, y) – v (x, y)} {\ Delta x}} \\ {} & = & {\ dfrac { \ partial u} {\ partial x} (x, y) + i \ dfrac {\ partial v} {\ partial x} (x, y)} \ end {array} \]

Вертикальное направление: \ (\ Delta x = 0 \), \ (\ Delta z = i \ Delta y \) (Мы сделаем это немного быстрее.)

\ [\ begin {array} {rcl} {f ‘(z)} & = & {\ lim _ {\ Delta z \ to 0} \ dfrac {f (z + \ Delta z) – f (z)} { \ Delta z}} \\ {} & = & {\ lim _ {\ Delta y \ to 0} \ dfrac {(u (x, y + \ Delta y) + iv (x, y + \ Delta y)) – (u (x, y) + iv (x, y))} {i \ Delta y}} \\ {} & = & {\ lim _ {\ Delta y \ to 0} \ dfrac {u (x, y + \ Дельта y) – u (x, y)} {i \ Delta y} + i \ dfrac {v (x, y + \ Delta y) – v (x, y)} {i \ Delta y}} \\ { } & = & {\ dfrac {1} {i} \ dfrac {\ partial u} {\ partial y} (x, y) + \ dfrac {\ partial v} {\ partial y} (x, y)} \ \ {} & = & {\ dfrac {\ partial v} {\ partial y} (x, y) – i \ dfrac {\ partial u} {\ partial y} (x, y)} \ end {array} \ ]

Мы нашли два различных представления \ (f ‘(z) \) в терминах частичных \ (u \) и \ (v \).Если сложить их вместе, мы получим уравнения Коши-Римана:

\ [f ‘(z) = \ dfrac {\ partial u} {\ partial x} + i \ dfrac {\ partial v} {\ partial x} = \ dfrac {\ partial v} {\ partial y} – i \ dfrac {\ partial u} {\ partial y} \ \ \ Rightarrow \ \ \ dfrac {\ partial u} {\ partial x} = \ dfrac {\ partial v} {\ partial y}, \ text {и} – \ dfrac {\ partial u} {\ partial y} = \ dfrac {\ partial v} {\ partial x}. \]

Оказывается, верно и обратное, и оно нам очень пригодится.

Теорема \ (\ PageIndex {2} \)

Рассмотрим функцию \ (f (z) = u (x, y) + iv (x, y) \), определенную в области \ (A \).Если \ (u \) и \ (v \) удовлетворяют уравнениям Коши-Римана и имеют непрерывные частичные, то \ (f (z) \) дифференцируема на \ (A \).

Проба

Доказательство этого – сложное упражнение в анализе. Это несколько выходит за рамки этого класса, поэтому мы его пропустим. Если вам интересно, приложив немного усилий, вы сможете понять это.

2.7.3 Использование уравнений Коши-Римана

Уравнения Коши-Римана предоставляют нам прямой способ проверки дифференцируемости функции и вычисления ее производной.z. \ nonumber \]

Пример \ (\ PageIndex {2} \)

Используйте уравнения Коши-Римана, чтобы показать, что \ (f (z) = \ overline {z} \) не дифференцируемо.

Раствор

\ (f (x + iy) = x – iy \), поэтому \ (u (x, y) = x, v (x, y) = -y \). Принятие частных производных

\ (u_x = 1 \), \ (u_y = 0 \), \ (v_x = 0 \), \ (v_y = -1 \)

Поскольку \ (u_x \ ne v_y \), уравнения Коши-Римана не выполняются и, следовательно, \ (f \) не дифференцируемо.

Теорема \ (\ PageIndex {3} \)

Если \ (f (z) \) дифференцируема на диске и \ (f ‘(z) = 0 \) на диске, то \ (f (z) \) постоянно.

Проба

Поскольку \ (f \) дифференцируема и \ (f ‘(z) \ Equiv 0 \), уравнения Коши-Римана показывают, что

\ [u_x (x, y) = u_y (x, y) = v_x (x, y) = v_y (x, y) = 0 \ nonumber \]

Мы знаем из многомерного исчисления, что функция от \ ((x, y) \) с обеими частями, равными тождественному нулю, является постоянной.Таким образом, \ (u \) и \ (v \) постоянны, а значит, и \ (f \).

2.7.4 \ (f ‘(z) \) как матрица \ (2 \ times 2 \)

Напомним, что мы могли бы представить комплексное число \ (a + ib \) как матрицу \ (2 \ times 2 \)

\ [a + ib \ \ leftrightarrow \ \ begin {bmatrix} a & -b \\ b & a \ end {bmatrix}. \]

Теперь, если мы запишем \ (f (z \) через \ ((x, y) \), мы получим

\ [f (z) = f (x + iy) = u (x, y) + iv (x, y) \ \ leftrightarrow \ f (x, y) = (u (x, y), v (x , у)).\]

У нас

\ [f ‘(z) = u_x + iv_x, \]

, поэтому мы можем представить \ (f ‘(z) \) как

\ [\ begin {bmatrix} u_x & -v_x \\ v_x & u_x \ end {bmatrix}. \]

Используя уравнения Коши-Римана, мы можем заменить \ (- v_x \) на \ (u_y \) и \ (u_x \) на \ (v_y \), что дает нам представление

\ [f ‘(z) \ \ leftrightarrow \ \ begin {bmatrix} u_x & u_y \\ v_x & v_y \ end {bmatrix}, \]

т.е. \ (f ‘(z) \) – это просто якобиан \ (f (x, y) \).

Мне легче запомнить якобиан, чем уравнения Коши-Римана. Поскольку \ (f ‘(z) \) – комплексное число, я могу использовать матричное представление в уравнении 1, чтобы запомнить уравнения Коши-Римана!

Производные более сложных функций

Немного более сложные функции

Программа SL не требует от студентов умения выводить следующие производные. Они есть в справочнике по формулам.2 (x) $… возможности почти безграничны. Есть несколько основных правил, которые позволяют вам иметь дело с этими более сложными функциями, не прибегая к использованию «основных принципов».

Правило суммы / разницы

Функция может быть определена как сумма (или разность) двух или более других функций, например $ f (x) = g (x) + h (x) $. Не потребуется слишком много усилий, чтобы из первых принципов показать, что производная этой функции равна:

(1)

\ begin {уравнение} f ‘(x) = g’ (x) + h ‘(x) \ end {уравнение}

или в нотации Лейбница:

(2)

\ begin {align} \ frac {d} {dx} f (x) = \ frac {d} {dx} g (x) + \ frac {d} {dx} h (x) \ end {align}

Этого нет в буклете с формулами IB, но мы все надеемся, что вы помните.

Правило продукта

Функцию можно определить как произведение двух (или более) функций, например $ f (x) = g (x) h (x) $. Приложив некоторые усилия и сообразительность, можно показать, что производная произведения двух функций равна:

(3)

\ begin {уравнение} f ‘(x) = g (x) h’ (x) + h (x) g ‘(x) \ end {уравнение}

или в нотации Лейбница:

(4)

\ begin {align} \ frac {d} {dx} f (x) = g (x) \ frac {d} {dx} h (x) + h (x) \ frac {d} {dx} g ( x) \ end {align}

Правило частного

Функцию можно определить как частное двух функций, например $ f (x) = \ frac {g (x)} {h (x)}] $.2} \ end {align}

Правило цепочки

Последний часто оказывается самым сложным. Это включает в себя поиск производной сложной функции. Например, функция $ f (x) = g (h (x)) $. Производная сложной функции:

(7)

\ begin {уравнение} f ‘(x) = g’ (h (x)) h ‘(x) \ end {уравнение}

Словами, это означает, что производная равна производной «внешней» функции, вычисленной «внутренней» функцией, умноженной на производную внутренней функции.

Да, это полный рот.В нотации Лейбница правило цепочки может выглядеть совсем иначе. Если мы определим:

(8)

\ begin {уравнение} y = g (u) \ end {уравнение}

с

(9)

\ begin {уравнение} u = f (x) \ end {уравнение}

Обратите внимание, что $ y = g (f (x)) $ просто старая добрая составная функция. Тогда мы можем сказать, что:

(10)

\ begin {align} \ frac {dy} {dx} = \ frac {dy} {du} \ frac {du} {dx} \ end {align}

Я, честно говоря, нахожу их обоих немного запутанными. Чтобы справиться с любым способом написания правила, просто потребуется немного практики.

Примеры

Есть предложения, например проблемы? Разместите их в комментариях ниже.


Хотите добавить или прокомментировать эти заметки? Сделайте это ниже.

% PDF-1.6 % 115 0 объект > / Outlines 35 0 R / Metadata 153 0 R / AcroForm 149 0 R / Pages 114 0 R / StructTreeRoot 1 0 R / Type / Catalog / Lang (en) >> эндобдж 35 0 объект > эндобдж 153 0 объект > поток 2008-02-19T19: 05: 21-05: 00JstorPdfGenerator v1.02008-10-11T03: 10: 33-07: 002008-10-11T03: 10: 33-07: 00iText 2.0.6 (от lowagie.com) application / pdfuuid: 6ed148f7-5cef-42b3-86ae-cbdfca714cccuuid: 7976f5f6- кафе-41ef-b8e9-521e5d86d8c7 конечный поток эндобдж 149 0 объект > / Кодировка >>>>> эндобдж 114 0 объект > эндобдж 1 0 obj > / Тип / StructTreeRoot >> эндобдж 148 0 объект > эндобдж 49 0 объект > / ProcSet [/ PDF / Text / ImageB / ImageC / ImageI] >> / Type / Page >> эндобдж 101 0 объект > эндобдж 100 0 объект > поток х + г 26S070 ճ 4 SIr * T0T0

3.2. Комплексные числа – Справочная документация по теоретической физике 0.5

 def arg (x):
    "" "
    Функция аргумента.
    "" "
    из журнала импорта cmath
    вернуть журнал (x) .imag

def generate_values ​​():
    "" "
    Создайте значения для проверки функции.
    "" "
    из математического импорта sin, cos, pi
    из случайного импорта случайный
    # Создайте 3 круга в комплексной плоскости с диаметрами 0,5, 1 и 2. Мы
    # избегайте специальных значений, таких как -1, +/- i и т. д., потому что они обычно отправляют
    # числовые значения, близкие к сечению ветви, и числовые ошибки, затем
    # переверните знак, e.грамм.:
    # В [1]: sqrt ((- 0.5j) ** 2)
    # Out [1]: -0,5j
    #
    # В [2]: (-0,5j) ** 2
    # Out [2]: (-0.25-0j)
    #
    # В [3]: sqrt (-0,25)
    # Out [3]: 0,5j
    # Здесь [3] - правильное значение, а [1] - неверное, но это происходит из-за
    # к ошибкам округления в [2] (малая отрицательная мнимая часть составляет
    # sqrt () возвращает -0,5j вместо + 0,5j).
    #
    # По этой причине мы выбрали N = 7.
    N = 7
    круг = []
    для n в диапазоне (N): circle.append (cos (2 * pi * n / N) + 1j * sin (2 * pi * n / N))
    values ​​= []
    для n в диапазоне (N): значения.добавить (0,5 * круг [n])
    для n в диапазоне (N): values.append (1.0 * circle [n])
    для n в диапазоне (N): values.append (2,0 * круг [n])
    # Добавьте несколько случайных точек:
    для n в диапазоне (30):
        values.append ((random () - 0,5) * 20 + 1j * (random () - 0,5) * 20)
    возвращаемые значения
значения = generate_values ​​()

def feq (a, b, max_relative_error = 1e-12, max_absolute_error = 1e-12):
    "" "
    Возвращает True, если a == b для заданных относительных и абсолютных ошибок, в противном случае
    Ложь.
    "" "
    # если числа достаточно близки (абсолютно), то они равны
    если abs (a-b)  abs (a):
        относительная_ошибка = абс (а-б) / абс (б)
    еще:
        относительная_ошибка = абс (а-б) / абс (а)
    #print abs (a-b), relative_error
    вернуть relative_error <= max_relative_error

def test_zero1 (слева, справа):
    "" "
    Проверяет, что комплексная функция f (x) одной комплексной переменной равна нулю."" "
    для x в значениях:
        r = feq (lhs (x), rhs (x))
        если не r:
            print "x lhs (x) rhs (x)"
            напечатать x, lhs (x), rhs (x)
            утверждать ложно

def test_zero2 (слева, справа):
    "" "
    Проверяет, что комплексная функция двух комплексных переменных f (x, y) равна нулю.
    "" "
    для x в значениях:
        для y в значениях:
            r = feq (lhs (x, y), rhs (x, y))
            если не r:
                print "x y lhs (x, y) rhs (x, y)"
                напечатать x, y, lhs (x, y), rhs (x, y)
                утверждать ложно

def test_zero3 (слева, справа):
    "" "
    Проверяет, что комплексная функция f (x, y, z) трех комплексных переменных равна нулю."" "
    для x в значениях:
        для y в значениях:
            для z в значениях:
                r = feq (lhs (x, y, z), rhs (x, y, z))
                если не r:
                    print "x y z lhs (x, y, z) rhs (x, y, z)"
                    напечатайте x, y, z, lhs (x, y, z), rhs (x, y, z)
                    утверждать ложно

def diff (f, z0, theta, eps = 1e-8):
    h = eps * exp (I * тета)
    return (f (z0 + h) -f (z0)) / h

def diff2 (dfdz, dfdconjz, z0, theta):
    вернуть dfdz (z0) + dfdconjz (z0) * exp (-2 * I * theta)

def test_zero (f, dfdz, dfdconjz, z0, theta, eps = 1e-8):
    утверждать feq (diff (f, z0, theta, eps), diff2 (dfdz, dfdconjz, z0, theta),
            max_relative_error = eps * 1e2, max_absolute_error = eps * 1e2)

от пола импорта математики, пи
из cmath импорт sqrt, exp, log
I = 1j

углы = [0, пи / 7, пи / 4, пи / 2, 3 * пи / 4, пи]

для x в значениях:
    для теты в углах:
        test_zero (лямбда x: abs (x), лямбда x: x.сопряженный () / (2 * абс (х)),
                лямбда x: x / (2 * abs (x)), x, theta)
        test_zero (лямбда x: журнал (x), лямбда x: 1 / x, лямбда x: 0, x, тета)
        test_zero (лямбда x: журнал (exp (x-x.conjugate ())), лямбда x: 1,
                лямбда x: -1, x, тета)
        test_zero (лямбда x: sqrt (x ** 2), лямбда x: sqrt (x ** 2) / x, лямбда x: 0, x, тета)
        test_zero (лямбда x: sqrt (x ** 2), лямбда x: x / sqrt (x ** 2), лямбда x: 0, x, тета)
 

Анализ | математика | Британника

Анализ , раздел математики, который имеет дело с непрерывными изменениями и с некоторыми общими типами процессов, которые возникли в результате изучения непрерывных изменений, таких как пределы, дифференциация и интеграция.С момента открытия Исааком Ньютоном и Готфридом Вильгельмом Лейбницем дифференциального и интегрального исчисления в конце 17 века анализ превратился в огромную и центральную область математических исследований с приложениями во всех науках и в таких областях, как финансы, экономика. , и социология.

Британская викторина

Определить: математические термины

Вот ваша миссия, если вы решите принять ее: Определите следующие математические термины до того, как истечет время.

Исторические истоки анализа можно найти в попытках вычислить пространственные величины, такие как длина кривой линии или площадь, ограниченная кривой. Эти проблемы могут быть сформулированы исключительно как вопросы математической техники, но они имеют гораздо более широкое значение, поскольку обладают широким разнообразием интерпретаций в физическом мире. Например, площадь внутри кривой представляет прямой интерес для измерения земли: сколько акров содержит участок земли неправильной формы? Но этот же метод также определяет массу однородного листа материала, ограниченного некоторой выбранной кривой, или количество краски, необходимое для покрытия поверхности неправильной формы.Менее очевидно, что эти методы можно использовать для определения общего расстояния, пройденного транспортным средством, движущимся с различной скоростью, глубины, на которой корабль будет плавать, когда он будет помещен в море, или общего расхода топлива ракеты.

Точно так же математический метод нахождения касательной к кривой в заданной точке может также использоваться для расчета крутизны изогнутого холма или угла, на который движущаяся лодка должна повернуть, чтобы избежать столкновения. Менее прямо это связано с чрезвычайно важным вопросом расчета мгновенной скорости или других мгновенных скоростей изменения, таких как охлаждение теплого объекта в холодной комнате или распространение болезнетворного организма через человеческую популяцию.

Эта статья начинается с краткого введения в исторические основы анализа и основные концепции, такие как системы счисления, функции, непрерывность, бесконечные ряды и пределы, которые необходимы для понимания анализа. После этого введения следует полный технический обзор, от расчетов до нестандартного анализа, а затем статья завершается полной историей.

Получите подписку Britannica Premium и получите доступ к эксклюзивному контенту. Подпишитесь сейчас

Историческая справка

Преодоление разрыва между арифметикой и геометрией

Математика делит явления на два широких класса, дискретные и непрерывные, исторически соответствующие разделению между арифметикой и геометрией.Дискретные системы можно подразделить только до сих пор, и их можно описать целыми числами 0, 1, 2, 3,…. Непрерывные системы можно подразделить до бесконечности, и для их описания требуются действительные числа, числа, представленные десятичными разложениями, такими как 3,14159…, возможно, продолжающимися бесконечно. Понимание истинной природы таких бесконечных десятичных знаков лежит в основе анализа.

Различие между дискретной математикой и непрерывной математикой является центральным вопросом для математического моделирования, искусства представления особенностей природного мира в математической форме.Вселенная не содержит реальных математических объектов и не состоит из них, но многие аспекты Вселенной очень похожи на математические концепции. Например, число два не существует как физический объект, но оно описывает важную особенность таких вещей, как человеческие близнецы и двойные звезды. Аналогичным образом действительные числа обеспечивают удовлетворительные модели для множества явлений, даже если никакая физическая величина не может быть измерена с точностью до более чем дюжины или около того десятичных знаков. К реальному миру применимы не значения бесконечного числа десятичных знаков, а дедуктивные структуры, которые они воплощают и активируют.

Анализ возник потому, что многие аспекты мира природы можно с успехом рассматривать как непрерывные - по крайней мере, с превосходной степенью приближения. Опять же, это вопрос моделирования, а не реальности. Материя на самом деле не непрерывна; если материю разделить на достаточно мелкие части, то появятся неделимые компоненты или атомы. Но атомы чрезвычайно малы, и для большинства приложений рассмотрение материи как континуума вносит незначительную ошибку и значительно упрощает вычисления.Например, моделирование континуума является стандартной инженерной практикой при изучении потока текучих сред, таких как воздух или вода, изгиба упругих материалов, распределения или потока электрического тока и потока тепла.

Открытие исчисления и поиск оснований

Два основных шага привели к созданию анализа. Первым было открытие удивительной связи, известной как фундаментальная теорема исчисления, между пространственными задачами, включающими вычисление некоторого общего размера или значения, например длины, площади или объема (интегрирование), и проблемами, связанными со скоростью изменения такие как наклон касательных и скоростей (дифференциация).Заслуга независимого открытия около 1670 г. фундаментальной теоремы исчисления вместе с изобретением методов ее применения принадлежит совместно Готфриду Вильгельму Лейбницу и Исааку Ньютону.

В то время как полезность исчисления в объяснении физических явлений была очевидна сразу, его использование бесконечности в вычислениях (посредством разложения кривых, геометрических тел и физических движений на бесконечное множество мелких частей) вызвало повсеместное беспокойство. В частности, англиканский епископ Джордж Беркли опубликовал знаменитую брошюру The Analyst; или «Беседа, адресованная неверному математику» (1734), в которой указывается, что исчисление - по крайней мере, в том виде, в каком его представили Ньютон и Лейбниц, - обладает серьезными логическими ошибками.Анализ вырос из кропотливого и тщательного изучения ранее нечетко определенных понятий, таких как функция и предел.

Подход Ньютона и Лейбница к исчислению был в основном геометрическим, включая отношения с «почти нулевыми» делителями - «флюксии» Ньютона и «бесконечно малые» Лейбница. В течение 18-го века исчисление становилось все более алгебраическим, поскольку математики - в первую очередь швейцарский Леонард Эйлер и итальянский француз Жозеф-Луи Лагранж - начали обобщать концепции непрерывности и пределов от геометрических кривых и тел до более абстрактных алгебраических функций и начали распространять эти идеи в комплексные числа.Хотя эти разработки не были полностью удовлетворительными с фундаментальной точки зрения, они были фундаментальными для возможного уточнения строгой основы для расчетов французом Огюстен-Луи Коши, богемцем Бернхардом Больцано и, прежде всего, немцем Карлом Вейерштрассом в 19 веке.

Оставить комментарий